History: Art & Culture

History: india before independence, history: india after independence, history: world, social science: caste, religion, region, globalization, social science: poverty, population, globalization, social science: globalization, social science: women, geography: physical, geography: climate, disaster related, geography: resources distribution, geography: factors affecting industrial locations, geography: urbanization / society: urbanization, appendix: linear paper of 2023-gsm1, appendix: model answer pe free lecture & powerpoint, appendix: syllabus of upsc mains-gsm1 general studies paper1.

In 2013, UPSC changed the syllabus-pattern of Mains examination and the number of general studies papers were increased from two to four. Out of them, GS Paper-I deals with History, Culture, Society and Geography. Overall breakup looks like this

Topicwise analysis of UPSC Mains GS Paper1 by Mrunal Graph

[Block-1] History

Topicwise analysis of UPSC Mains GS Paper1 by Mrunal Graph

GS1 Syllabus Topic: Salient aspects of Art, Architecture, literature from Ancient to Modern Times

How will you explain that medieval Indian temple sculptures represent the social life of those days ? (स्पष्ट करें कि मध्यकालीन भारतीय मंदिरों की मूर्तिकला उस दौर के सामाजिक जीवन का प्रतिनिधित्व करती है। )

GS1 Syllabus Topic: Mid-18th century – Present (significant events, personalities, issues); Freedom Struggle (various stages, important contributors from different parts of the country)

GS1 Syllabus Topic: Post-Independence (consolidation and reorganisation within country)

GS1 Syllabus Topic: 18th century events (e.g. Industrial revolution, WWs, redrawn boundaries, colonisation, decolonisation); Political philosophies (e.g. communism, capitalism, socialism) and their effect on society

[Block-2] Social Science

Topicwise analysis of UPSC Mains GS Paper1 by Mrunal Graph

GS1 Syllabus Topic:

  • Communalism, Regionalism, Secularism; Social Empowerment
  • Salient features of Indian Society; Diversity of India;

GS1 Syllabus Topic: Poverty, Population; Development and associated issues

GS1 Syllabus Topic: Globalisation (effects on Indian society)

  • GS1 Syllabus Topic: Role of women and women’s organisation;
  • Although some of the following questions fall under population and globalization categories, but if a person cultivates habit of noting down women related topics under one head, it’ll benefit in both GS1, GS2 and Essay.

[Block-3] Geography

Topicwise analysis of UPSC Mains GS Paper1 by Mrunal Graph

GS1 Syllabus Topic: Salient Features of World Physical Geography; Important Geophysical phenomena (earthquakes, tsunami, volcanoes, cyclones); Geographical features and location;

GS1 Syllabus Topic: Critical geographical features, flora, fauna (changes and effects thereof)

GS1 Syllabus Topic: Distribution of key Natural Resources (world, S. Asia, Indian subcontinent)

GS1 Syllabus Topic: Factors responsible for location of Industries (primary, secondary, tertiary; India, world)

GS1 Syllabus Topic: Urbanization: problems and remedies

  • Explain the role of geographical factors towards the development of Ancient India. (प्राचीन भारत के विकास के लिए भौगोलिक कारकों की भूमिका को स्पष्ट करें।) (150 words/10m)
  • What are the main features of Vedic society and religion? Do you think some of the features are still prevailing in Indian society? (वैदिक समाज और धर्म की मुख्य विशेषताएं क्या हैं? क्या आपको लगता है कि भारतीय समाज में कुछ विशेषताएं अभी भी प्रचलित हैं?) (250 words/15m)
  • What were the major technological changes introduced during the Sultanate period? How did those technological changes influence the Indian society? (सल्तनत युग के दौरान प्रमुख तकनीकी परिवर्तन क्या पेश किए गए थे? उन तकनीकी परिवर्तनों ने भारतीय समाज को कैसे प्रभावित किया?) (250 words/15m)
  • What was the difference between Mahatma Gandhi and Rabindranath Tagore in their approach towards education and nationalism? (शिक्षा और राष्ट्रवाद के प्रति उनके दृष्टिकोण में महात्मा गांधी और रबींद्रनाथ टैगोर के बीच क्या अंतर था?) (150 words/10m)
  • How did the colonial rule affect the tribals in India and what was the tribal response to the colonial oppression? (औपनिवेशिक शासन ने भारत में आदिवासियों को कैसे प्रभावित किया और औपनिवेशिक उत्पीड़न के लिए आदिवासी प्रतिक्रिया क्या थी?) (250 words/15m)
  • From being net food importer in 1960s, India has emerged as a net food exporter to the world. Provide reasons. (1960 के दशक में ख्याधआयातक होने से, भारत दुनिया के लिए खाद्य निर्यातक के रूप में उभरा है। कारण प्रदान करें।) (250 words/15m)
  • Bring out the socio-economic effects of the introduction of railways in different countries of the world (दुनिया के विभिन्न देशों में रेलवे की शुरूआत के सामाजिक-आर्थिक प्रभावों को बाहर लाएं) (150 words/10m)
  • Why did human development fail to keep pace with economic development in India? (भारत में आर्थिक विकास के साथ मानव विकास क्यों विफल रहा?) (250 words/15m)
  • Does urbanization lead to more segregation and/or marginalization of the poor in Indian metropolises? (क्या शहरीकरण भारतीय महानगरों में गरीबों के अधिक अलगाव और/या हाशिए पर ले जाता है?) (250 words/15m)
  • Do you think marriage as a sacrament is loosing its value in Modern India? (क्या आपको लगता है कि एक संस्कार के रूप में विवाह आधुनिक भारत में अपना मूल्य खो रहा है?) (150 words/10m)
  • Explain why suicide among young women is increasing in Indian society. (बताएं कि भारतीय समाज में युवा महिलाओं के बीच आत्महत्या क्यों बढ़ रही है।) (150 words/10m)
  • Child cuddling is now being replaced by mobile phones. Discuss its impact on the socialization of children. (बच्चों को दुलारने की जगह अब मोबाइल फोन द्वारा प्रतिस्थापित किया जा रहा है। बच्चों के समाजीकरण पर इसके प्रभाव पर चर्चा करें।) (150 words/10m)
  • Why is caste identity in India both fluid and static? (भारत में जाति की पहचान द्रव और स्थिर दोनों क्यों है?) (250 words/15m)
  • Discuss the impact of post-liberal economy on ethnic identity and communalism. (जातीय पहचान और सांप्रदायिकता पर उदारवादी अर्थव्यवस्था के प्रभाव पर चर्चा करें।) (250 words/15m)
  • Discuss the consequences of climate change on the food security in tropical countries. (उष्णकटिबंधीय देशों में खाद्य सुरक्षा पर जलवायु परिवर्तन के परिणामों पर चर्चा करें।) (150 words/10m)
  • Why is the world today confronted with a crisis of availability of and access to freshwater resources? (आज दुनिया को मीठे पानी के संसाधनों की उपलब्धता और पहुंच के संकट के साथ क्यों सामना किया जाता है?) (150 words/10m)
  • Identify and discuss the factors responsible for diversity of natural vegetation in India. Assess the significance of wildlife sanctuaries in rain forest regions of India (भारत में प्राकृतिक वनस्पति की विविधता के लिए जिम्मेदार कारकों की पहचान करें और चर्चा करें। भारत के वर्षा वन क्षेत्रों में वन्यजीव अभयारण्यों के महत्व का आकलन करें) (250 words/15m)
  • How are the fjords formed? Why do they constitute some of the most picturesque areas of the world? (Fjords कैसे बनते हैं? वे दुनिया के कुछ सबसे सुरम्य क्षेत्रों का गठन क्यों करते हैं?) (150 words/10m)
  • Why is the South-West Monsoon called Purvaiya’ (easterly) in Bhojpur Region? How has this directional seasonal wind system influenced the cultural ethos of the region? (भोजपुर क्षेत्र में दक्षिण-पश्चिम मानसून को पुरवाया (ईस्टरली) क्यों कहा जाता है? इस दिशात्मक मौसमी पवन प्रणाली ने क्षेत्र के सांस्कृतिक लोकाचार को कैसे प्रभावित किया है?) (150 words/10m)
  • Comment on the resource potentials of the long coastline of India and highlight the status of natural hazard preparedness in these areas. (भारत की लंबी तटरेखा की संसाधन क्षमता पर टिप्पणी करें और इन क्षेत्रों में प्राकृतिक खतरे की तैयारियों की स्थिति को उजागर करें।) (250 words/15m)

Appendix: Linear paper of 2022-GSM1

  • How will you explain that medieval Indian temple sculptures represent the social life of those days ? (स्पष्ट करें कि मध्यकालीन भारतीय मंदिरों की मूर्तिकला उस दौर के सामाजिक जीवन का प्रतिनिधित्व करती है। ) 10m 150w
  • Why did the armies of the British East India Company – mostly comprising of Indian soldiers – win consistently against the more numerous and better equipped armies of the then Indian rulers ? Give reasons. (अधिकांश भारतीय सिपाहियों वाली ईस्ट इंडिया की सेना क्यों तत्कालीन भारतीय शासकों की संख्याबल में अधिक और बेहतर सुसज्जित सेना से लगातार जीतती रही ? कारण बताएँ ।) 10m 150w
  • Why was there a sudden spurt in famines in colonial India since the mid-eighteenth century ? Give reasons. (औपनिवेशिक भारत की अठारहवीं शताब्दी के मध्य से क्यों अकाल पड़ने में अचानक वृद्धि देखने को मिलती है? कारण बताएं।) 10m 150w
  • Describe the characteristics and types of primary rocks. (प्राथमिक चट्टानों की विशेषताओं एवं प्रकारों का वर्णन कीजिए।) 10m 150w
  • Discuss the meaning of colour-coded weather warnings for cyclone prone areas given by India Meteorological Department. (भारतीय मौसम विज्ञान विभाग द्वारा चक्रवात प्रवण क्षेत्रों के लिए मौसम सम्बन्धी चेतावनियों के लिए निर्धारित रंग-संकेत के अर्थ की चर्चा करें।) 10m 150w
  • Discuss the natural resource potentials of ‘Deccan Trap’. ( ‘दक्कन ट्रैप’ की प्राकृतिक संसाधन सम्भावनाओं की चर्चा कीजिए।) 10m 150w
  • Examine the potential of wind energy in India and explain the reasons for their limited spatial spread. (भारत में पवन ऊर्जा की संभावना का परीक्षण कीजिए एवं उनके सीमित क्षेत्रीय विस्तार के कारणों को समझाइए|) 10m 150w
  • Explore and evaluate the impact of ‘Work From Home’ on family relationships. (पारिवारिक सम्बन्धों पर ‘वर्क फ्रॉम होम’ के असर की छानबीन तथा मूल्यांकन करें ।) 10m 150w
  • How is the growth of Tier 2 cities related to the rise of a new middle class with an emphasis on the culture of consumption ? (उपभोक्ता संस्कृति के विशेष परिप्रेक्ष्य में नव मध्यवर्ग के उभार से टीयर 2 शहरों का विकास किस तरह सम्बन्धित है ?) 10m 150w
  • Given the diversities among tribal communities in India, in which specific contexts should they be considered as a single category ? (भारत के जनजातीय समुदायों की विविधताओं को देखते हुए किस विशिष्ट सन्दर्भ के अन्तर्गत उन्हें किसी एकल श्रेणी में माना जाना चाहिए ?) 10m 150w
  • The political and administrative reorganization of states and territories has been a continuous ongoing process since the mid-nineteenth century. Discuss with examples. (राज्यों एवं प्रदेशों का राजनीतिक और प्रशासनिक पुनर्गठन उन्नीसवीं शताब्दी के मध्य में निरंतर चल रही एक प्रक्रिया है। उदाहरण सहित विचार करें।) 15m 250w
  • Discuss the main contributions of Gupta period and Chola period to Indian heritage and culture. (भारतीय परम्परा और संस्कृति में गुप्त-काल और चोल काल के योगदान पर चर्चा करें।) 15m 250w
  • Discuss the significance of the lion and bull figures in Indian mythology, art and architecture. (भारतीय मिथक कला और वास्तुकला में सिंह एवं वृषभ की आकृतियों के महत्व पर विचार करें।) 15m 250w
  • What are the forces that influence ocean currents? Describe their role in fishing industry of the world. (समुद्री धाराओं को प्रभावित करने वाली शक्तियाँ कौन सी है ? विश्व के मत्स्य उद्योग में इनके योगदान का वर्णन करें ) 15m 250w
  • Describing the distribution of rubber producing countries, indicate the major environmental issues faced by them. (रबर उत्पादक देशों के वितरण का वर्णन करते हुए उनके द्वारा सामना किए जाने वाले प्रमुख पर्यावरणीय मुद्दों को इंगित कीजिए।) 15m 250w
  • Mention the significance of straits and isthmus in international trade. (अंतर्राष्ट्रीय व्यापार में जलसंधि व स्थलसंधि के महत्त्व का उल्लेख कीजिए ।.) 15m 250w
  • Troposphere is a very significant atmospheric layer that determines weather processes. How? ( क्षोभमंडल वायुमंडल का एक महत्त्वपूर्ण परत है जो मौसम प्रक्रियाओं को निर्धारित करता है। कैसे ?) 15m 250w
  • Analyse the salience of ‘sect’ in Indian society vis-a-vis caste, region and religion. (भारतीय समाज में जाति, क्षेत्र तथा धर्म के समानांतर ‘पंथ’ की विशेषता की विवेचना कीजिए ।) 15m 250w
  • Are tolerance, assimilation and pluralism the key elements in the making of an Indian form of secularism ? Justify your answer. (क्या सहिष्णुता, सम्मिलन एवं बहुलता मुख्य तत्त्व हैं जो धर्मनिरपेक्षता के भारतीय रूप का निर्माण करते हैं ? तर्कसंगत उत्तर दें।) 15m 250w
  • Elucidate the relationship between globalization and new technology in a world of scarce resources, with special reference to India. (अपर्याप्त संसाधनों की दुनिया में भूमंडलीकरण एवं नए तकनीक के रिश्ते को भारत के विशेष सन्दर्भ में स्पष्ट करें। ) 15m 250w
  • Free Lecture: Youtube.com/playlist?list=PLSVABdpsEeglA4VimijYd4u1Q0uee8Vpz
  • Free Powerpoint: https://unacademy.com/content/upsc/mains-paper-analysis/
  • 1) Salient aspects of Art, Architecture,
  • 2) literature from Ancient to Modern Times
  • Mid-18th century – Present (significant events, personalities, issues); Freedom Struggle (various stages, important contributors from different parts of the country)
  • Post-Independence (consolidation and reorganisation within country)
  • 18th century events (e.g. Ind’l revo, WWs, redrawn boundaries, colonisation, decolonisation); Political philosophies (e.g. communism, capitalism, socialism) and their effect on society
  • Poverty, Population; Development and associated issues
  • Salient features of Indian Society; Diversity of India; Globalisation (effects on Indian society)
  • Role of women and women’s organisation;
  • Critical geographical features, flora, fauna (changes and effects therof)
  • Factors responsible for location of Industries (primary, secondary, tertiary; India, world)
  • Salient Features of World Physical Geography; Important Geophysical phenomena (earthquakes, tsunami, volcanoes, cyclones); Geographical features and location;
  • Distribution of key Natural Resources (world, S. Asia, Indian subcontinent)
  • Urbanisation: problems and remedies

Arihant General Studies Paper-1 PDF [2022 Edition]

Karthik M

  • April 3, 2023
  • Education , Pavithran.Net , UPSC Exams

If you’re an aspiring civil servant gearing up for the UPSC Prelims, you’re probably already aware of the importance of having the right study material. As the competition gets tougher, it becomes crucial to have access to the best resources available. That’s where the Arihant General Studies Manual Paper 1 Pdf comes in. The 2022 edition of this comprehensive guidebook is a must-have for anyone preparing for the UPSC Prelims.

Table of Contents

Arihant General Studies Paper 1 Pdf Download

With its detailed coverage of all the topics in the General Studies Paper 1 syllabus, this book is designed to help you ace the exam. From Indian history, geography, and polity to economics, environment, and current affairs, the Arihant General Studies Manual has it all. So, whether you’re a first-time aspirant or a seasoned candidate, this book is an indispensable tool that can help you crack the UPSC Prelims with flying colors. In this article, we’ll take a closer look at what makes the Arihant General Studies Manual so special and why you should consider adding it to your study arsenal.

Arihant General Studies Manual Paper 1 Pdf

The General Studies Paper – 1 by Arihant is a highly popular book that is specifically created for the civil services Preliminary exams. It comprises 6 main sections that cover the entire UPSC syllabus. Additionally, there is a dedicated section that covers current affairs, such as conferences and summits. The language used in the book is simple and easy to understand, allowing readers to grasp the concepts with ease. The book also includes 5 sets of practice questions, with topic-wise questions and solved papers from previous years. Overall, the “General Studies Manual Paper 1” is a comprehensive package that guarantees success, providing updated and authentic content in adherence to the prescribed pattern.

Arihant General Studies Manual Paper-1 - Dr. Priya Goel Pdf

The book covers traditional subjects such as History, Geography, Polity, and General Science, which have been updated regularly. It also includes questions arranged by chapters and sections. In addition, contemporary topics like Indian Economy, Environment & Ecology, Science & Technology, and General Awareness have been explained with the latest facts and figures to help readers understand the concepts better. The latest national and international events have been listed in a separate section.

To cater to the exam’s trend, the book includes practice sets at the end. Finally, the attached booklet provides over 5000 essential points for revision, making it a valuable tool that serves as a one-stop solution for the Civil Services Preliminary Examination preparation.

Book Details:-

  • Complete Package for UPSC aspirants
  • Excellent compilation of Content using easy language
  • Current affairs could be outdated in this Book.

Inside this Book:

  • Solved Paper 2021-2018
  • Indian History and Indian National Movement
  • India and World Geography
  • Indian Polity and Governance
  • Indian Economy
  • General Science & Science and Technology
  • General Knowledge & Computer Technology
  • Practice: Topicewise Questions
  • Current Affairs
  • Crack Sets (1-5)

Arihant General Studies Manual Paper-1 2023 - Manohar Pandey

General Studies Manual Paper-1 2023 for Civil Services Preliminary Exam

  • Latest Edition: 11 th (2023)
  • No of Pages: 1648

28 Years UPSC CSE Prelims Solved Papers 1 & 2 (1995-2022)

28 Years UPSC CSE Prelims Solved Papers 1 & 2 (1995-2022)

Introducing the 28 Years UPSC CSE Prelims Solved Papers 1 & 2 (1995-2022) by IAS Manish Rannjan. This comprehensive book is the ultimate solution to all your UPSC CSE Prelims preparation woes.

  • Language: English
  • Edition: March 2021
  • Pdf Quality: Original

Useful Books for Competitive Exams

  • Download Adda247 Books PDF
  • Download Kiran Books PDF
  • Download Arihant Books PDF
  • Download History Books PDF
  • Download English Books PDF
  • Download Aptitude Books PDF
  • Download GK & General Awareness Books PDF
  • Download Geography Books PDF
  • Download Economics Books PDF
  • Download Polity Books PDF
  • Download Banking Books PDF
  • Download SSC Exam Books PDF
  • Download Disha Books PDF

Best Books for Competitive Exams [PDF]

Disclaimer: Pavithran.Net doesn’t aim to promote or condone piracy in any way. We do not own any of these books. We neither create nor scan this Book. The Images, Books & other Contents are copyrighted to their respective owners. We are providing PDFs of Books that are already available on the Internet, Websites, and Social Media like Facebook, Telegram, Whatsapp, etc. We highly encourage visitors to Buy the Original content from their Official Sites. If any way it violates the law or if anybody has Copyright issues/ having discrepancies over this post, Please Take our Contact Us page to get in touch with us. We will reply as soon as we receive your Mails.

We Need Your Support. Please Share the Link if it is helpful to your Cherished circle

Related Posts

ForumIAS Set of 5 Workbooks for UPSC Prelims PDF

ForumIAS Workbooks for UPSC Prelims PDF

  • March 24, 2024

Uma Kapila Indian Economy Pdf

Uma Kapila Indian Economy PDF [Latest Edition]

  • March 20, 2024

Indian Economy - Vivek Singh PDF

[PDF] Indian Economy Vivek Singh | 7th Edition (2023-24)

GC Leong Certificate Physical & Human Geography PDF

GC Leong Certificate Physical & Human Geography PDF

Arihant Cracking the CSAT Paper 2 PDF Download

Arihant Cracking the CSAT Paper 2 PDF [11th Edition]

  • March 19, 2024

Upkar's General Science PDF

Upkar’s General Science PDF

Leave a reply cancel reply.

Your email address will not be published. Required fields are marked *

Name  *

Email  *

Add Comment  *

Save my name, email, and website in this browser for the next time I comment.

Post Comment

  • IAS Preparation
  • UPSC Preparation Strategy
  • UPSC Prelims 2022 gs Question Paper 1

UPSC Prelims GS Question Paper 1 2022 - Download PDF

Every year, the Union Public Service Commission holds the Civil Services exam to fill a number of vacancies in All India Services and several Central Civil Services. The Civil Services Examination is the most prestigious and competitive examination of the UPSC. This year, the IAS Prelims Exam  concluded on June 5, 2022 .

Aspirants can download the UPSC 2022 Prelims Answer Key in the linked article. 

The official IAS notification giving details about the exam was released on Feb 2, 2022. The commission released the admit card for the exam on May 10, 2022.

The Question Paper of UPSC Prelims GS 1 2022 is available to download. The aspirants can  download the PDF of the IAS Prelims 2022 General Studies-I Question Paper, using the link, given below.

The Prelims exam result will be declared after a month of the exam, the eligible candidates who would have cleared the Prelims 2022 will have to fill DAF-I to appear in UPSC Mains 2022 which commenced from September 16,  2022, onwards.

UPSC IAS Prelims 2022 GS 1 Question Paper

UPSC Prelims GS (Paper 1) Question Paper 2022 PDF:- Download PDF Here The exam pattern of GS Paper 1 is given in the following table:

It is to be noted that the prelims examination is used as a screening test for applicants to select for the mains exam. The scores obtained at this stage are not considered for the final merit list, however, students must prepare thoroughly for this exam because cut-offs are unpredictable and rely on the average score each year.

UPSC GS Paper 1

The UPSC CSE Preliminary exam comprises of two papers that are conducted on the same day.

  • UPSC Prelims General Studies Paper 1
  • UPSC Prelims General Studies Paper 2 (CSAT)

UPSC Prelims 2022 GS Paper 2 is qualifying in nature, i.e, candidates need to score a minimum of 33% of the total marks of the paper, where scores of UPSC Prelims 2022 GS Paper 1 determine the cut-off and decides whether a candidate has qualified the first stage of the exam to write the UPSC Mains.

UPSC Prelims GS Paper 1 Cut-off Marks for Previous years

UPSC Prelims GS Paper 1 Marking Scheme:

As mentioned above, the first stage of the exam is qualifying in nature and marks scored in the exam are not counted in the final merit list. The marking scheme of IAS Prelims GS1 Question Paper 2022 is given below:

  • General Studies Paper 1 consists of a total of 100 questions for 200 marks .
  • Each question carries 2 marks .
  • For every right answer , candidates are awarded +2 marks .
  • For every wrong answer , 1/3rd of the total marks allotted to that question are deducted, i.e, -0.66 marks are given to the candidates for every wrong answer.
  • For example : If a candidate has attempted all 100 questions, out of which 70 are right and 30 are wrong the total score of the candidate will be: (70×2)- (30×0.66) = 120.20 marks.

To get details of the UPSC admit card for the different stages of the examination, aspirants can check out the linked article.

UPSC Prelims GS Paper 1: Subjects

The main subjects from which the questions are usually asked in the UPSC Prelims exam are:

  • Indian Polity and Governance – Constitution, Political System, Panchayati Raj, Public Policy, Rights Issues, etc.
  • History – History of India and Indian National Movement.
  • Indian and World Geography – Physical, Social, Economic Geography of India and the World.
  • Economic and Social Development – Sustainable Development, Poverty, Inclusion, Demographics, Social Sector initiatives, etc.
  • Environmental Ecology – General issues on Environmental Ecology, Biodiversity and Climate Change – that do not require subject specialisation
  • Current affairs – Current events of national and international importance.
  • General Science- Latest topics related to science and technology.

Aspirants can go through the detailed UPSC Prelims Syllabus , to get a better understanding of the exam pattern and to comprehend the standard of examination.

For current affairs, further, visit:

  • Latest Current Affairs
  • Daily News Analysis of The Hindu Newspaper
  • Best of PIB 
  • BYJU’S IAS Youtube Channel

Most Important Topics for UPSC Prelims 2023

IAS Exam syllabus is very vast and therefore aspirants must learn to grasp and observe any current event of national and worldwide importance through the prism of the UPSC syllabus and relevance to the IAS exam. For performing well in the exam, it is a must to cover the static as well as the advanced syllabus of the exam. Given below, we have provided links to important topics that must be covered for the UPSC CSE 2023 Prelims General Studies Question Paper 1 :

Current Affairs

  • List of Important Current Affairs Topics for UPSC 2022
  • Current Affairs questions for UPSC Prelims
  • Current Affairs topics for 2021
  • Union Budget 2022
  • Highlights of Economic Survey 2022

Candidates looking for subject-specific questions from the previous year’s prelims examination, can visit the topic-wise UPSC prelims previous year question papers with answers page and get a segregated set of papers for each of the IAS subject.For any further exam updates, study material and preparation tips, turn to BYJU’S for assistance.

Leave a Comment Cancel reply

Your Mobile number and Email id will not be published. Required fields are marked *

Request OTP on Voice Call

Post My Comment

general studies manual paper 1 2022 pdf

IAS 2024 - Your dream can come true!

Download the ultimate guide to upsc cse preparation.

  • Share Share

Register with BYJU'S & Download Free PDFs

Register with byju's & watch live videos.

general studies manual paper 1 2022 pdf

  • Exam Preparation
  • Exams by UPSC

general studies manual paper 1 2022 pdf

Download the free Kindle app and start reading Kindle books instantly on your smartphone, tablet or computer – no Kindle device required .

Read instantly on your browser with Kindle for Web.

Using your mobile phone camera, scan the code below and download the Kindle app.

QR code to download the Kindle App

Follow the author

Rakesh Kumar Roshan

Image Unavailable

General Studies Manual Paper-1 2022 for Civil Serives Preliminary Exam

  • To view this video download Flash Player

General Studies Manual Paper-1 2022 for Civil Serives Preliminary Exam Paperback – 20 December 2021

Save extra with 2 offers.

  • Free Delivery

10 days Replacement

  • Amazon Delivered
  • Pay on Delivery
  • Secure transaction

Replacement Instructions

general studies manual paper 1 2022 pdf

Purchase options and add-ons

"1. General Studies Paper – 1 is the best- selling book particularly designed for the civil services Preliminary examinations. 2. This book is divided into 6 major sections covering the complete syllabus as per UPSC pattern 3. Special Section is provided for Current Affairs covering events, Summits and Conferences 4. simple and lucid language used for better understanding of concepts 5. 5 Crack Sets are given for practice 6. Practice Questions provides Topicwise Questions and Previous Years’ Solved Papers

With our all time best selling edition of “General Studies Manual Paper 1” is a guaranteed success package which has been designed to provide the complete coverage to all subjects as per prescribed pattern along with the updated and authentic content. The book provides the conventional Subjects like History, Geography, Polity and General Science that are thoroughly updated along with Chapterwise and Sectionwise questions. Contemporary Topics likes; Indian Economy, Environment & Ecology, Science & Technology and General Awareness have also been explained with latest facts and figures to ease the understanding about the concepts in this book. Current events of national and international interest have been listed in a separate section. Practice Sets are given at the end, keeping in view the trend of the questions coming in exams. Lastly, More than 5000 Most Important Points for Revision are provided in the attached booklet of the guide. It is a must have tool that proves to be one point solution for the preparf Civil Services Preliminary Examination.

Solved Paper 2021-2018, Indian History and Indian National Movement, India and World Geography, Indian Polity and Governance, Indian Economy, General Science & Science and Technology, General Knowledge & Computer Technology, Practice: Topicewise Questions, Current Affairs, Crack Sets (1-5)."

  • ISBN-10 9325796090
  • ISBN-13 978-9325796096
  • Edition Tenth
  • Publisher Arihant Publications
  • Publication date 20 December 2021
  • Language English
  • Dimensions 20.3 x 25.4 x 4.7 cm
  • Print length 1648 pages
  • See all details

Frequently bought together

General Studies Manual Paper-1 2022 for Civil Serives Preliminary Exam

Customers who viewed this item also viewed

Arihant Cracking The CSAT (Civil Services Aptitude Test) Paper-2

Product description

About the author, product details.

  • Publisher ‏ : ‎ Arihant Publications; Tenth edition (20 December 2021); Arihant Publications India Pvt. Ltd.
  • Language ‏ : ‎ English
  • Paperback ‏ : ‎ 1648 pages
  • ISBN-10 ‏ : ‎ 9325796090
  • ISBN-13 ‏ : ‎ 978-9325796096
  • Item Weight ‏ : ‎ 1 kg
  • Dimensions ‏ : ‎ 20.3 x 25.4 x 4.7 cm
  • Country of Origin ‏ : ‎ India
  • Packer ‏ : ‎ Arihant Eduweb
  • #236 in UPSC Civil Services Prelims

About the author

Rakesh kumar roshan.

Discover more of the author’s books, see similar authors, read author blogs and more

Customer reviews

Reviews with images.

Customer Image

  • Sort reviews by Top reviews Most recent Top reviews

Top reviews from India

There was a problem filtering reviews right now. please try again later..

general studies manual paper 1 2022 pdf

  • Press Releases
  • Amazon Science
  • Sell on Amazon
  • Sell under Amazon Accelerator
  • Protect and Build Your Brand
  • Amazon Global Selling
  • Become an Affiliate
  • Fulfilment by Amazon
  • Advertise Your Products
  • Amazon Pay on Merchants
  • COVID-19 and Amazon
  • Your Account
  • Returns Centre
  • 100% Purchase Protection
  • Amazon App Download
  • Conditions of Use & Sale
  • Privacy Notice
  • Interest-Based Ads
  • Skip to primary navigation
  • Skip to main content
  • Skip to primary sidebar

UPSC Coaching, Study Materials, and Mock Exams

Enroll in ClearIAS UPSC Coaching Join Now Log In

Call us: +91-9605741000

General Studies for Civil Services Preliminary Examination (Paper -1) by Pearson – Book Review

Last updated on September 26, 2023 by ClearIAS Team

General Studies

Table of Contents

General Studies for Civil Services Preliminary Examination (Combo Pack) by Pearson India: What’s special?

Pearson General Studies for Civil Services Preliminary Exam 2019

General Studies for Civil Services Preliminary Examination (Paper -1) by Pearson is a combo pack of 6 books covering almost the entire syllabus (at an MRP of just Rs.1599. Additional discount is available for students). This package contains 6 books – each with its own binding:

  • Book 1 – General Knowledge and Current Affairs
  • Book 2 – Indian Polity and Governance
  • Book 3 – Indian Economy – Economic and Social Development
  • Book 4 – Geography and Environmental Ecology
  • Book 5 – General Science
  • Book 6 – History and Culture

If you are a fresher who just start IAS exam preparation , it’s not advisable to go behind hundreds of books. That will deviate your attention from the fundamental topics which you should invariably cover. Your focus should be to get your basics right with the best resources. In that sense, we strongly recommend getting a copy of General Studies for Civil Services Preliminary Examination (Paper -1) by Pearson.

Key Highlights of General Studies Paper 1 for Civil Services Preliminary Examination

UPSC CSE General Studies Paper 1 Pearson

  • Simple, easy-to-read, and concise.
  • Due to separate binding, these books are very handy, unlike other all-in-one heavy GS manuals.
  • Macro and micro analysis of the UPSC Civil Service Exam Question paper trends – highly useful for beginners and veterans to understand the changing UPSC requirements and priority areas.
  • Supported by clear graphical illustrations, tables and charts.
  • Topic-wise solved multiple choice questions (MCQs).
  • Previous Year UPSC CSE Question Papers (2016, 2017, and 2018) with answers.
  • Two Online Mock Tests are provided FREE. You will get an online access code along with this book.
  • As an online bonanza, you would get – 10+ previous year’s papers tagged topic-wise, 70+ coloured maps, and additional questions for practice.
  • Cost-effective and high return on investment.

Note: For General Studies Paper -2 (CSAT), Pearson has another best-selling book. Click to buy it online

Authors: Edgar Thorpe and Showick Thorpe

Edgar Thorpe and Showick Thorpe are a father-son duo.

ClearIAS UPSC Coaching

Showick Thorpe is an expert on competitive examinations. He has authored more than 40 books in English, Hindi and Bangla languages, catering to various competitive examinations and career development.

About Pearson

Pearson is the world’s learning company, with a presence in 70 countries worldwide. Pearson’s products and services are spread across the Higher education sector as well as the K12 sector.

How can you buy ‘General Studies for Civil Services Preliminary Examination by Pearson?

Pearson GS Manuals 2019

  • General Studies for Civil Services Preliminary Examination (Paper -1) – Click to buy online from Amazon (English Edition)
  • General Studies for Civil Services Preliminary Examination (Paper -2) – Click to buy online from Amazon (English Edition)
  • General Studies for Civil Services Preliminary Examination (Paper -1) – Click to buy online from Amazon (Hindi Edition)

Print Friendly, PDF & Email

Review of 'General Studies for Civil Services Preliminary Examination (Paper -1) by Pearson'

UPSC CSE General Studies Paper 1 Pearson

Book Title: General Studies for Civil Services Preliminary Examination (Paper -1) by Pearson

Book Description: General Studies for Civil Services Preliminary Examination (Paper -1) by Pearson is a combo pack of 6 books covering almost the entire syllabus of UPSC Civil Services Preliminary Exam.

Book Author: Edgar Thorpe and Showick Thorpe

Book Format: Paperback

ClearIAS UPSC Prelims Test Series

ISBN: 9353065704

Number Of Pages: 1968

  • Ease of Reading
  • Exam Relavance
  • Flowcharts, Pictures, and Maps
  • Coverage of UPSC Syllabus
  • Language and Presentation
  • Paper Quality

If you are a fresher starting your IAS preparation, Pearson's General Studies for Civil Services Preliminary Examination (Paper -1) is a great choice. As an all-in-one package with 6 different books (separate covers), it really serves as handy and affordable guide for IAS exam preparation. The book will keep you focused on the requirements of UPSC. A great value for money product!

  • Click to Buy General Studies for Civil Services Preliminary Examination (Paper -1) by Pearson

Aim IAS, IPS, or IFS?

ClearIAS UPSC Coaching

About ClearIAS Team

ClearIAS is one of the most trusted learning platforms in India for UPSC preparation. Around 1 million aspirants learn from the ClearIAS every month.

Our courses and training methods are different from traditional coaching. We give special emphasis on smart work and personal mentorship. Many UPSC toppers thank ClearIAS for our role in their success.

Download the ClearIAS mobile apps now to supplement your self-study efforts with ClearIAS smart-study training.

Reader Interactions

Leave a reply cancel reply.

Your email address will not be published. Required fields are marked *

Don’t lose out without playing the right game!

Follow the ClearIAS Prelims cum Mains (PCM) Integrated Approach.

Join ClearIAS PCM Course Now

UPSC Online Preparation

  • Union Public Service Commission (UPSC)
  • Indian Administrative Service (IAS)
  • Indian Police Service (IPS)
  • IAS Exam Eligibility
  • UPSC Free Study Materials
  • UPSC Exam Guidance
  • UPSC Prelims Test Series
  • UPSC Syllabus
  • UPSC Online
  • UPSC Prelims
  • UPSC Interview
  • UPSC Toppers
  • UPSC Previous Year Qns
  • UPSC Age Calculator
  • UPSC Calendar 2024
  • About ClearIAS
  • ClearIAS Programs
  • ClearIAS Fee Structure
  • IAS Coaching
  • UPSC Coaching
  • UPSC Online Coaching
  • ClearIAS Blog
  • Important Updates
  • Announcements
  • Book Review
  • ClearIAS App
  • Work with us
  • Advertise with us
  • Privacy Policy
  • Terms and Conditions
  • Talk to Your Mentor

Featured on

ClearIAS Featured in The Hindu

and many more...

general studies manual paper 1 2022 pdf

Take ClearIAS Mock Exams: Analyse Your Progress

ClearIAS Course Image

Analyse Your Performance and Track Your All-India Ranking

Ias/ips/ifs online coaching: target cse 2025.

ClearIAS Course Image

Are you struggling to finish the UPSC CSE syllabus without proper guidance?

BooksByBSF

GENERAL STUDIES Manual 2022 |GS Paper 1 | UPSC | Civil Services Prelim | Other State Examinations

  • ISBN: 9789354601477
  • Edition/Reprint: 2022
  • Author(s): Surender Singh
  • Publisher: MCGRAW HILL EDUCATION
  • Product ID: 590491
  • Country of Origin: India
  • Availability: Sold Out
  • DESCRIPTION

About Product

General Studies Paper I 2022 is a thoroughly updated and comprehensive study material with a dedicated focus on the latest syllabus prescribed by the UPSC for Preliminary Examination. This revised annual edition employs coverage of the contemporary issues and developments. There is a profound change in the nature of questions and questioning techniques, and due stress has been laid on analytical aspects by correlating core concepts with the events at global and national level. This edition has undergone immense structural changes, making the content more precise and examination centric by weeding out redundant information. The 2022 edition is a one-stop solution for the aspirants to begin preparation for the Civil Services Examination. Salient Fetaures : ? Comprehensive coverage of the syllabus of UPSC and State Public Service Examinations, as per the latest examination pattern ? Includes solved 2020 Preliminary examination question paper ? New content creation on Governance covering topics like Regulation of Social Media influencers, Electoral Reforms, Anti-Defection Law, Political Reforms, Contempt of Court and many more ? Covers all aspects of COVID-19, vaping epidemic, multi-drug resistance, Black Fungus Disease, Mechanism of SARS-CoV-2 coronavirus infection in Human Beings, etc. ? Economics and Social Development section includes topics like Impact of Covid 19 on Economic Development, HDR 2020, World Happiness Report 2021, Global Hunger Index (GHI), National Nutrition Mission, India Skills Report 2021, etc. ? History of India, Indian and World Geography, Environmental Ecology, Biodiversity and Climate Change, and Science and Technology have been thoroughly revised and restructured to mind maps, flow charts, tables and follow point-wise content presentation ? Enriched web supplement available on the link/QR code given below ? o Monthly current affairs o 2021 Preliminary question paper with solution and analysis o Subject-wise mock test papers

  • About Publisher

MC Grawhill

MC Grawhill

Mc Graw Hill offers learning materials in print and digital formats for higher education and test preparation materials. It offers quality content for UPSC, CAT, NEET, CTET/STET, CA, and JEE exams and for students of Calcutta University, Engineering, and Business & Economics. ‘Indian Polity by M Laxmikanth’, ‘Indian Economy by Ramesh Singh’, ‘The Living World Morphology and Anatomy by Dr. Subroto Biswas and Dr. Aman Biswas’, and ‘Financial Management by Anutam Pal’ are the most popular books by this firm. Click here to view all book of this publisher

Related Books

General Studies Paper-2

General Studies Paper-2

General Studies Paper-1

General Studies Paper-1

10000+ Objective General Studies Mcq 100% Explanatory Notes for Civil Services & other Competitive Exams

10000+ Objective General Studies Mcq 100% Explanatory Notes for Civil Services & other Competitive Exams

Samanya Adhyayan 1993-2022 (30 Varsh)

Samanya Adhyayan 1993-2022 (30 Varsh)

UPSC Prelims GS Paper-1 15 Privious Years Main Solved Papers (2008-22)

UPSC Prelims GS Paper-1 15 Privious Years Main Solved Papers (2008-22)

Erroless 13 Year Uppsc General Studies

Erroless 13 Year Uppsc General Studies

Samanya Adhayan Prashnapatra-I 2023

Samanya Adhayan Prashnapatra-I 2023

28 Years Ias (pre) Solved Paper General Studies

28 Years Ias (pre) Solved Paper General Studies

Master The Mains – General Studies Mains (GS Paper III): Solved Previous Years' and Model Questions | UPSC Civil Services Exam

Master The Mains – General Studies Mains (GS Paper III): Solved Previous Years' and Model Questions | UPSC Civil Services Exam

Master The Mains – General Studies Mains (GS Paper II): Solved Previous Years' and Model Questions | UPSC Civil Services Exam

Master The Mains – General Studies Mains (GS Paper II): Solved Previous Years' and Model Questions | UPSC Civil Services Exam

NCERT Samanya Adhyayan | Civil Seva/ Rajya Seva Ki Prarambhik Pariksha Ke Liye (Hindi Edition)

NCERT Samanya Adhyayan | Civil Seva/ Rajya Seva Ki Prarambhik Pariksha Ke Liye (Hindi Edition)

Master The Mains – General Studies Mains (GS Paper IV): Solved Previous Years' and Model Questions | UPSC Civil Services Exam

Master The Mains – General Studies Mains (GS Paper IV): Solved Previous Years' and Model Questions | UPSC Civil Services Exam

73562+ books.

Added in last 24 Hours

Daily Visitor

Brand Slider

BooksbyBSF

  • UPSC Mains PYQ (1979 to 2023)
  • UPSC Result
  • UPSC Syllabus
  • UPSC Interview
  • Art & Culture
  • Environment
  • International Relation
  • Previous Year Paper’s
  • Science & Tech
  • Toppers Copy
  • Agriculture Optional Notes
  • Anthropology Optional Notes
  • Chemistry Optional Notes
  • Commerce Optional Notes
  • Economics Optional Notes
  • Geography Optional Notes
  • History Optional Notes
  • Law Optional Notes
  • Mathematics Optional Notes
  • Philosophy Optional Notes
  • Public Administration Optional Notes
  • Political Science Optional Notes
  • Physics Optional Notes
  • Sociology Optional Notes
  • GS Score Prelims Test
  • Only IAS Prelims Test
  • Rau’s IAS Prelims Test
  • Shankar IAS Prelims Test
  • Vision IAS Prelims Test – English
  • Vision IAS Prelims Test – Hindi
  • Insight IAS – English
  • Insight IAS – Hindi
  • Next IAs Prelims Test
  • Vision Ias Mains Test – English
  • Vision Ias Mains Test – Hindi
  • Next IAS Mains Test
  • Rau’s IAS Mains Test
  • GS Score Mains Test
  • Insight IAS Mains Test – English
  • Insight IAS Mains Test – Hindi
  • Anthropology Optional Test
  • Geography Optional Test
  • Geology Optional Notes
  • History Optional Test
  • Mathematics Optional Test
  • Optional Test Series
  • PSIR Optional Test
  • Public Administration Optional Test
  • Sociology Optional Test
  • Vision IAS Monthly – English
  • Vision IAS Monthly – Hindi
  • GS Score Monthly
  • GS Score Weekly
  • Kurukshetra – English
  • Kurukshetra – Hindi
  • Rau’s IAS Monthly
  • Rau’s Prelims Compass
  • Rau’s Mains Compass
  • Yojana English
  • Yojana Hindi
  • Insight IAS Magazine – English
  • Insights IAS Magazine – Hindi
  • Vision IAS – English
  • Vision IAS – Hindi
  • Shankar IAS
  • Standard Books
  • NCERT Books
  • IGNOU Books
  • Sign in / Join

general studies manual paper 1 2022 pdf

UPSC CSE Mains 2022 GS Paper I: GENERAL STUDIES PAPER-I PDF Download

[ general studies paper – 1 ] : upsc mains civil services ias exam 2022 question paper pdf download.

The Mains General Studies Paper I of the UPSC Civil Services Examination is being organised in the forenoon session of 17th September 2022. In total, Civil Services (Main) Exam is composed of nine papers and the examination is held for a period of 5 days.

The GS Paper I contain the following subjects:

  • Indian Heritage and Culture
  • Geography of the World

UPSC 2022 Mains GS Paper 1

The General Studies Paper I consists of 20 descriptive type questions on topics such as Indian Heritage and Culture, History and Geography of the world as well as Society. This paper is worth 250 marks and has a time limit of 3 hours. This paper is being considered for merit ranking. It is also vital to remember that all of the questions are mandatory to answer.

📍 General Studies Paper – 1 Mark’s Analysis

1. Modern History: 35 2. Geography: 100 3. Society: 75 4. Art and culture: 40

Breakage 1. Art and culture : 1 (10 marks) , 2 (15 Marks) 2. Modern History: 2 questions (10 marks) and 1question (15 marks) 3. Society: 3 question (10 marks) and 3 question (15 marks) 4. Geography : 4 question (10 marks) and 4 question (15 marks)

UPSC Mains 2022 GS Paper I :-   Download PDF Here

general studies manual paper 1 2022 pdf

GENERAL STUDIES

  • How will you explain that medieval Indian temple sculptures represent the social life of those days? (Answer in 150 words)
  • Why did the armies of the British East India Company – mostly comprising of Indian soldiers – win consistently against the more numerous and better equipped armies of the then Indian rulers? Give reasons. (Answer in 150 words)
  • Why was there a sudden spurt in famines in colonial India since the mid-eighteenth century? Give reasons. (Answer in 150 words)
  • Describe the characteristics and types of primary rocks. (Answer in 150 words)
  • Discuss the meaning of colour-coded weather warnings for cyclone prone areas given by India Meteorological Department. (Answer in 150 words)
  • Discuss the natural resource potentials of ‘Deccan Trap`. (Answer in 150 words)
  • Examine the potential of wind energy in India and explain the reasons for their limited spatial spread (Answer in 150 words)
  • Explore and evaluate the impact of ‘Work From Home’ on family relationships. (Answer in 150 words)
  • How is the growth of Tier 2 cities related to the rise of a new middle class with an emphasis on the culture of consumption? (Answer in 150 words)
  • Given the diversities among tribal communities in India, in which specific contexts should they be considered as a single category? (Answer in 150 words)
  • The political and administrative reorganization of states and territories has been a continuous ongoing process since the mid-nineteenth century. Discuss with examples. (Answer in 250 words)
  • Discuss the main contributions of Gupta period and Chola period to Indian heritage and culture. (Answer in 250 words)
  • Discuss the significance of the lion and bull figures in Indian mythology, art and architecture. (Answer in 250 words)
  • What are the forces that influence ocean currents? Describe their role in fishing industry of the world. (Answer in 250 words)
  • Describing the distribution of rubber producing countries, indicate the major environmental issues faced by them. (Answer in 250 words)
  • Mention the significance of straits and isthmus in international trade. (Answer in 250 words)
  • Troposphere is a very significant atmospheric layer that determines weather process. How? (Answer in 250 words)
  • Analyse the salience of ‘sect’ in Indian society vis-a-vis caste, region and religion (Answer in 250 words)
  • Are tolerance, assimilation, and pluralism the key elements in the making of Indian form of secularism Justify your answer. (Answer in 250 words)
  • Elucidate the relationship between globalization and new technology in a world of scarce resources, with special reference to India. (Answer in 250 words)

UPSC Mains 2022 – General Studies 1 Question Paper

This section would be updated as soon as UPSC CSE (Mains) GS 1 paper is concluded.

  • Advertisement
  • Privacy Policy

Javascript not detected. Javascript required for this site to function. Please enable it in your browser settings and refresh this page.

general studies manual paper 1 2022 pdf

Call us @ 08069405205

general studies manual paper 1 2022 pdf

Search Here

general studies manual paper 1 2022 pdf

  • An Introduction to the CSE Exam
  • Personality Test
  • Annual Calendar by UPSC-2024
  • Common Myths about the Exam
  • About Insights IAS
  • Our Mission, Vision & Values
  • Director's Desk
  • Meet Our Team
  • Our Branches
  • Careers at Insights IAS
  • Daily Current Affairs+PIB Summary
  • Insights into Editorials
  • Insta Revision Modules for Prelims
  • Current Affairs Quiz
  • Static Quiz
  • Current Affairs RTM
  • Insta-DART(CSAT)
  • Insta 75 Days Revision Tests for Prelims 2024
  • Secure (Mains Answer writing)
  • Secure Synopsis
  • Ethics Case Studies
  • Insta Ethics
  • Weekly Essay Challenge
  • Insta Revision Modules-Mains
  • Insta 75 Days Revision Tests for Mains
  • Secure (Archive)
  • Anthropology
  • Law Optional
  • Kannada Literature
  • Public Administration
  • English Literature
  • Medical Science
  • Mathematics
  • Commerce & Accountancy
  • Monthly Magazine: CURRENT AFFAIRS 30
  • Content for Mains Enrichment (CME)
  • InstaMaps: Important Places in News
  • Weekly CA Magazine
  • The PRIME Magazine
  • Insta Revision Modules-Prelims
  • Insta-DART(CSAT) Quiz
  • Insta 75 days Revision Tests for Prelims 2022
  • Insights SECURE(Mains Answer Writing)
  • Interview Transcripts
  • Previous Years' Question Papers-Prelims
  • Answer Keys for Prelims PYQs
  • Solve Prelims PYQs
  • Previous Years' Question Papers-Mains
  • UPSC CSE Syllabus
  • Toppers from Insights IAS
  • Testimonials
  • Felicitation
  • UPSC Results
  • Indian Heritage & Culture
  • Ancient Indian History
  • Medieval Indian History
  • Modern Indian History
  • World History
  • World Geography
  • Indian Geography
  • Indian Society
  • Social Justice
  • International Relations
  • Agriculture
  • Environment & Ecology
  • Disaster Management
  • Science & Technology
  • Security Issues
  • Ethics, Integrity and Aptitude

InstaCourses

  • Indian Heritage & Culture
  • Enivornment & Ecology

Print Friendly, PDF & Email

[ GENERAL STUDIES PAPER – 1 ] : UPSC MAINS CIVIL SERVICES IAS EXAM 2022 QUESTION PAPER

DOWNLOAD ESSAY QUESTION PAPER : UPSC CSE MAINS 2022

DOWNLOAD General Studies 2 QUESTION PAPER : UPSC CSE MAINS 2022

DOWNLOAD General Studies 3 QUESTION PAPER : UPSC CSE MAINS 2022

DOWNLOAD General Studies 4 QUESTION PAPER : UPSC CSE MAINS 2022

UPSC MAINS GENERAL STUDIES PAPER – 1  MAINS 2022 

GENERAL STUDIES

1. How will you explain the medieval Indian temple sculptures represent the social life of those days? (Answer in 150 words) 10

2. Why did the armies of the British East India Company – mostly comprising of Indian soldiers – win consistently against the more numerous and better equipped armies of the Indian rulers? Give reasons. (Answer in 150 words) 10

3. Why was there a sudden spurt in famines in colonial India since the mid-eighteenth century? Give reasons. (Answer in 150 words) 10

4. Describe the characteristics and types of primary rocks. (Answer in 150 words) 10

5. Discuss the meaning of colour-coded weather warnings for cyclone prone areas given by India Meteorological department. (Answer in 150 words) 10

6. Discuss the natural resource potentials of ‘Deccan trap’. (Answer in 150 words) 10

7. Examine the potential of wind energy in India and explain the reasons for their limited spatial spread. (Answer in 150 words) 10

8. Explore and evaluate the impact of ‘Work From Home’ on family relationships. (Answer in 150 words) 10

9. How is the growth of Tier 2 cities related to the rise of a new middle class with an emphasis on the culture of consumption? (Answer in 150 words) 10

10. Given the diversities among the tribal communities in India, in which specific contexts should they be considered as a single category? (Answer in 150 words) 10

11. The political and administrative reorganization of states and territories has been a continuous ongoing process since the mid-nineteenth century. Discuss with examples. (Answer in 250 words) 15

12. Discuss the main contributions of Gupta period and Chola period to Indian heritage and culture . (Answer in 250 words) 15

13. Discuss the significance of the lion and bull figures in Indian mythology, art and architecture. (Answer in 250 words) 15

14. What are the forces that influence ocean currents? Describe their role in fishing industry of the world. (Answer in 250 words) 15

15. Describing the distribution of rubber producing countries, indicate the major environmental issues faced by them. (Answer in 250 words) 15

16. Mention the significance of straits and isthmus in international trade. (Answer in 250 words) 15

17. Troposphere is a very significant atmospheric layer that determines weather processes. How? (Answer in 250 words) 15

18. Analyze the salience of ‘sect’ in Indian society vis-a-vis caste, region and religion. (Answer in 250 words) 15

19. Are tolerance, assimilation and pluralism the key elements in the making of an Indian form of secularism? Justify your answer. (Answer in 250 words) 15

20. Elucidate the relationship between globalization and new technology in a world of scarce resources, with special reference to India. (Answer in 250 words) 15

general studies manual paper 1 2022 pdf

Check the recently released flagship course of InsightsIAS  – IPM 4.0 , IPM+ & IPM ++ Here

general studies manual paper 1 2022 pdf

  • Follow us on our Official  TELEGRAM  Channel  HERE
  • Subscribe to Our Official YouTube Channel   HERE
  • Our Official Facebook Page  HERE
  • Follow us on Twitter HERE
  • Follow our  Instagram ID   HERE
  • Subscribe to Our Podcast Channel   HERE

Left Menu Icon

  • Our Mission, Vision & Values
  • Director’s Desk
  • Commerce & Accountancy
  • Previous Years’ Question Papers-Prelims
  • Previous Years’ Question Papers-Mains
  • Environment & Ecology
  • Science & Technology
  • TEST SERIES

Material For Exam

  • __gs paper 1
  • __gs paper 2
  • __gs paper 3
  • __gs paper 4
  • __political science
  • __sociology
  • __economics
  • __geography
  • _OTHER BANK EXAMINATIONS
  • Current Affair
  • _Daily Current Affairs
  • _Monthly Current Affairs
  • _Yearly Current Affairs
  • _Dainik Jagaran National edition
  • _Indian Express Adfree
  • _THE HINDU NEWSPAPER IMPORTANT ARTICLES
  • _the hindu notes
  • _EMPLOYMENT NEWS
  • _EDITORIALS IN ENGLISH
  • _EDITORIALS IN HINDI
  • notification

Recent Update

Saturday, september 17, 2022, upsc cse mains 2022 gs paper i: general studies paper-i pdf download.

 UPSC CSE Mains 2022 GS Paper I: GENERAL STUDIES PAPER-I PDF Download

GENERAL STUDIES

PAPER – 1

1. How will you explain the medieval Indian temple sculptures represent the social life of those days? (Answer in 150 words) 10

2. Why did the armies of the British East India Company – mostly comprising of Indian soldiers – win consistently against the more numerous and better equipped armies of the Indian rulers? Give reasons. (Answer in 150 words) 10

3. Why was there a sudden spurt in famines in colonial India since the mid-eighteenth century? Give reasons. (Answer in 150 words) 10

4. Describe the characteristics and types of primary rocks. (Answer in 150 words) 10

5. Discuss the meaning of colour-coded weather warnings for cyclone prone areas given by India Meteorological department. (Answer in 150 words) 10

6. Discuss the natural resource potentials of ‘Deccan trap’. (Answer in 150 words) 10

7. Examine the potential of wind energy in India and explain the reasons for their limited spatial spread. (Answer in 150 words) 10

8. Explore and evaluate the impact of ‘Work From Home’ on family relationships. (Answer in 150 words) 10

9. How is the growth of Tier 2 cities related to the rise of a new middle class with an emphasis on the culture of consumption? (Answer in 150 words) 10

10. Given the diversities among the tribal communities in India, in which specific contexts should they be considered as a single category? (Answer in 150 words) 10

11. The political and administrative reorganization of states and territories has been a continuous ongoing process since the mid-nineteenth century. Discuss with examples. (Answer in 250 words) 15

12. Discuss the main contributions of Gupta period and Chola period to Indian heritage and culture . (Answer in 250 words) 15

13. Discuss the significance of the lion and bull figures in Indian mythology, art and architecture. (Answer in 250 words) 15

14. What are the forces that influence ocean currents? Describe their role in fishing industry of the world. (Answer in 250 words) 15

15. Describing the distribution of rubber producing countries, indicate the major environmental issues faced by them. (Answer in 250 words) 15

16. Mention the significance of straits and isthmus in international trade.  (Answer in 250 words)  15

17. Troposphere is a very significant atmospheric layer that determines weather processes. How? (Answer in 250 words) 15

18. Analyze the salience of ‘sect’ in Indian society vis-a-vis caste, region and religion. (Answer in 250 words) 15

19. Are tolerance, assimilation and pluralism the key elements in the making of an Indian form of secularism? Justify your answer. (Answer in 250 words) 15

20. Elucidate the relationship between globalization and new technology in a world of scarce resources, with special reference to India. (Answer in 250 words) 15

general studies manual paper 1 2022 pdf

SEARCH CONTENTS

Recent posts, facebook page, contact form.

Please enable JavaScript! Bitte aktiviere JavaScript! S'il vous plaît activer JavaScript! Por favor,activa el JavaScript! antiblock.org

general studies manual paper 1 2022 pdf

Wonderslate login

The Joy of Learning

Login failed. Please try again!

New User? Sign-up

Verification code is sent to

Did not receive the OTP? Resend

Already registered? Login

Forgot Password

Don't worry. We'll fix this for you!

Verification code has been sent to

Wonderslate Logo

  • Current Affairs
  • LeaderBoard
  • Register for Free

Competitive Exams books

Competitive Exams text books, online test series and study materials

eBooks Filter Icon

Smart eBook Benefits

ebook Feature Read Anywhere, Anytime

Read Anywhere, Anytime

Discover the freedom of reading your eBooks offline with Wonderslate Smart eBooks. No more lugging around heavy books – now you can enjoy your favorite reads on the go, whether you're commuting, traveling, or just relaxing at home.

ebook Feature Study Smarter, Not Harder

Study Smarter, Not Harder

Ace your exams with Wonderslate Smart eBooks as your study buddy. Easily highlight crucial points like definitions and formulas, making revising a breeze. Say goodbye to flipping through countless pages and juggling bookmarks – your study materials are at your fingertips, organized and ready for review.

ebook Feature Create Your Personal Booklet

Create Your Personal Booklet

Revolutionize your note-taking experience by creating a personalized booklet of your most important notes and highlights. Now, you can effortlessly carry your revision essentials to school or college, streamlining your study routine with automated revision lists.

ebook Feature Seamless Reading on Any Device

Seamless Reading on Any Device

Whether you prefer a larger screen or the convenience of your mobile phone, Wonderslate Smart eBooks ensures a seamless reading experience. Our Android and desktop apps offer you the flexibility to read anytime, anywhere. Metro rides or cozy corners – your eBooks are just a click away.

ebook Feature Achieve Your Exam Goals with Ease

Achieve Your Exam Goals with Ease

Wonderslate goes beyond just reading. Track your progress, spend time efficiently with online test preparations, and witness your results unfold. Tailored to meet your examination goals, Wonderslate unlocks a plethora of features designed to make your learning journey smooth and successful.

ebook Feature Seamless Reading on Any Device

Effortless Progress Tracking

Keep tabs on your reading time and progress effortlessly. Wonderslate allows you to measure the time invested in your eBooks, take online test preparations, and view your results. It's your personalized toolkit to conquer your exam goals – all in one place.

For bulk and library orders contact us at [email protected] or 8088443860

eBook is added to your cart!

Online test series + ebook.

[email protected]

  • Our Centers Delhi Bhubaneswar Lucknow

general studies manual paper 1 2022 pdf

Free Resources

  • UPSC Previous Year Papers

UPSC IAS Prelims 2022 GS Paper 1 With Solution

  • Categories 2022
  • Published 17th Sep, 2022
  • Download PDF

img

UPSC Prelims Exam 2022 was held on 5th, June 2022 (Sunday). As the official answer sheet is released by UPSC  once the final result comes out. In this scenario, all aspirants need an authentic and reliable source for making a fine assessment of marks in the  IAS Prelims exam . This assessment is quite necessary as it helps you to decide the contours of your preparation strategy.   The students remain quite apprehensive regarding their selection in prelims due to the uncertain trend of cutoff in UPSC prelims.  Students who remain on the fringe of expected cut-off are often haunted by the result of UPSC Prelims.

Keeping the student’s situation in cognizance , GS SCORE  has once again come up with  the  most reliable, authentic, and accurate  Prelims Answer Key  of  UPSC Prelims 2022 . Aspirants will be able to access  IAS Prelims 2022   Answer Key just after a few hours of Prelims General Studies Paper 1 . The Link for  UPSC Prelims Answer Key 2022 will be activated after a few hours of Paper 1(General Studies).

Candidates can calculate their estimated score using the  Prelims Answer Key . The official  PT Answer Key 2022  will be released soon after the  UPSC Prelims  are conducted.

UPSC IAS Prelims Answer Key 2022  will help students to know their correct and incorrect responses and they can calculate the marks that they are going to score in the Prelims exam. The final selection of the candidates for the IAS post depends on the marks secured in the UPSC Mains exam and Interview. UPSC Prelims Exam is the first step of the selection Process but the marks will not be considered while preparing the Final Merit List.

The UPSC Mains 2022 exam is scheduled to begin on October 16, 2022. The UPSC will release the UPSC Prelims 2022 Answer Key  on its official website. All those candidates who will appear for the exam can check the  IAS Prelims  Answer Key 2022  on the official website.

The candidates are informed that the various coaching institutes release the  Civil Services Prelims Answer key  on the same day of the exam. However, UPSC will release the Official answer keys once the Recruitment process is over.

1. “Rapid Financing Instrument” and “Rapid Credit Facility” are related to the provisions of lending by which one of the following?

(a) Asian Development Bank

(b) International Monetary Fund

(c) United Nations Environment Programme Finance Initiative

(d) World Bank

Correct Option: (b) Explanation:

Rapid Financing Instrument (RFI)

The Rapid Financing Instrument (RFI) provides rapid financial assistance, which is available to all member countries facing an urgent balance of payments need. The RFI was created as part of a broader reform to make the IMF’s financial support more flexible to address the diverse needs of member countries. The RFI replaced the IMF’s previous emergency assistance policy and can be used in a wide range of circumstances.

Rapid Credit Facility (RCF)

The Rapid Credit Facility (RCF) provides rapid concessional financial assistance to low-income countries (LICs) facing an urgent balance of payments (BoP) need with no ex post conditionality where a full-fledged economic program is neither necessary nor feasible. The RCF was created under the Poverty Reduction and Growth Trust (PRGT) as part of a broader reform to make the Fund’s financial support more flexible and better tailored to the diverse needs of LICs, including in times of crisis. There are three windows under RCF: (i) a “regular window” for urgent BoP needs caused by wide range of sources including domestic instability, emergencies and fragility; (ii) an “exogenous shock window” for urgent BoP needs caused by a sudden, exogenous shock; and (iii) a “large natural disaster window” for urgent BoP needs arising from natural disasters where damage is assessed to be equivalent to or exceed 20 percent of the member’s GDP. Access under the RCF is subject to annual and cumulative limits, with higher access limits applying for the large natural disaster window. For higher income countries that are non-PRGT eligible, a similar Rapid Financing Instrument (RFI) is available.

Source: https://www.imf.org/en/About/Factsheets/Sheets/2016/08/02/19/55/Rapid-Financing-Instrument

https://www.imf.org/en/About/Factsheets/Sheets/2016/08/02/21/08/Rapid-Credit-Facility

2. With reference to the Indian economy, consider the following statements:

  • An increase in Nominal Effective Exchange Rate (NEER) indicates the appreciation of rupee.
  • An increase in the Real Effective Exchange Rate (REER) indicates an improvement in trade competitiveness.
  • An increasing trend in domestic inflation relative to inflation in other countries is likely to cause an increasing divergence between NEER and REER.

Which of the above statements are correct?

(a) 1 and 2 only

(b) 2 and 3 only

(c) 1 and 3 only

(d) 1, 2 and 3

Correct Option: (c) Explanation:

  • Statement 1 is correct: NEER is a measure of value of a currency against a weighted average of several foreign currency. An increase in NEER indicates appreciation of rupee.
  • Statement 2 is incorrect: An increase in REER implies that exports become more expensive and imports become cheaper; therefore, an increase indicates a loss in trade competitiveness.
  • Statement 3 is correct: NEER is the weighted geometric average of the bilateral nominal exchange rates of the home currency in terms of foreign currencies. The REER is the weighted average of NEER adjusted by the ratio of domestic price to foreign prices. Increasing trend in domestic inflation relative to inflation in other countries creates a divergence in NEER and REER.
  • Reserve Bank of India - RBI Bulletin
  • What is real effective exchange rate (REER)? – IMF DATA Help

3. With reference to the Indian economy, consider the following statements:

  • If the inflation is too high, Reserve Bank of India (RBI) is likely to buy government securities.
  • If the rupee is rapidly depreciating, RBI is likely to sell dollars in the market.
  • If interest rates in the USA or European Union were to fall, that is likely to induce RBI to buy dollars.

Which of the statements given above are correct?

  • Statement 1 is incorrect: If the inflation is high RBI tries to reduce the liquidity from the market, by selling Government securities to the public via open market operation.
  • Statement 2 is correct: Rupee depreciation means, fall in value of rupee with respect to dollar. In free floating exchange rate regime, depreciation takes place when the demand for dollar is more than the supply.thus, RBI is likely to sell dollars in the economy to increase the supply of the dollar.
  • Statement 3 is correct: If the interest rate in US and EU falls, there will be an inflow of dollars in the Indian market, leading to appreciation of the rupee. To reduce the supply of dollar in the economy, RBI will like to buy the dollars from the market.

4. With reference to the “G20 Common Framework”, consider the following statements:

  • It is an initiative endorsed by the G20 together with the Paris Club.
  • It is an initiative to support Low Income Countries with unsustainable debt.

Which of the statements given above is/are correct?

(c) Both 1 and 2

(d) Neither 1 nor 2

  • Statement 1 is correct: Initiative endorsed by G20 together with Paris Club.
  • Statement 2 is correct: It is an initiative to support low income countries with unsustainable debt.

The G20 Common Framework for Debt Treatments Must Be Stepped Up – IMF Blog

5. With reference to the India economy, what are the advantages of “Inflation-Indexed Bonds (IIBs)”?

  • Government can reduce the coupon rates on its borrowing by way of IIBs.
  • IIGs provide protection to the investors from uncertainty regarding inflation.
  • The interest received as well as capital gains on IIBs are not taxable.

Correct Option: (a) Explanation:

  • Statement 1 is correct: Since these bonds provide no risk of capital loss, it can offer a lesser rate of interest (coupon) as interest is directly proportional to risk.
  • Statement 2 is correct: Inflation indexed bonds provide protection to investors from uncertainty regarding inflation.
  • Statement 3 is incorrect : Interest or inflation compensation both are taxable. There is no special treatment for these bonds.

Source: Reserve Bank of India - Frequently Asked Questions (rbi.org.in)

6. With reference to foreign-owned e-commerce firms operating in India, which of the following statements is/are correct?

  • They can sell their own goods in addition to offering their platforms as market-places.
  • The degree to which they can own big sellers on their platforms is limited.

Select the correct answer using the code given below:

  • Statement 1 is correct:    E- commerce firms can sell their own products in addition to offering their platforms as market place.
  • Statement 2 is correct:   Big sellers have the limit of 25% for sale on e-commerce platform

Source:   E-Commerce Laws and Regulations in India - Vakilsear

7. Which of the following activities constitute real sector in the economy?

  • Farmers harvesting their crops
  • Textile mills converting raw cotton into fabrics
  • A commercial bank lending money to a trading company
  • A corporate body issuing Rupee Denominated Bonds overseas

(b) 2, 3 and 4 only

(c) 1, 3 and 4 only

(d) 1, 2, 3 and 4

The real sector of the economy deals with the production side, while the nominal economy deals with the financial side. Financial activities majorly support real(production) activity, but does not contribute itself too much except the factor income it generates.

8. Which one of the following situations best reflects “Indirect Transfers” often talked about in media recently with reference to India?

(a) An Indian company investing in a foreign enterprise and paying taxes to the foreign country on the profits arising out of its investment

(b) A foreign company investing in India and paying taxes to the country of its base on the profits arising out of its investment

(c) An Indian company purchases tangible assets in a foreign country and sells such assets after their value increases and transfers the proceeds to India

(d) A foreign company transfers shares and such shares derive their substantial value from assets located in India

Correct Option: (d) Explanation:

  • Indirect transfers refer to situations where when foreign entities own shares or assets in India, the shares of such foreign entities are transferred instead of a direct transfer of the underlying assets in India.

Source:   TAXI-451008 665..676 (nishithdesai.com)

9. With reference to the expenditure made by an organization or a company, which of the following statements is/are correct?

  • Acquiring new technology is capital expenditure.
  • Debt financing is considered capital expenditure, while equity financing is considered revenue expenditure.
  • Statement 1 is correct:  Acquiring new technology is considered as capital expenditure as it will generate profit in the future and helps in creation of new assets.
  • Statement 2 is incorrect: Debt Financing and equity financing are considered under capital expenditure.

10. With reference to the Indian economy, consider the following statements:

  • A share of the household financial savings goes towards government borrowings.
  • Dated securities issued at market-related rates in auctions form a large component of internal debt.

Which of the above statements is/are correct?

  • Statement 1 is correct: A share of household financial savings goes to the government borrowings, as part of public accounts of India. It mainly consists of provident funds.
  • Statement 2 is correct: Dated securities means regular government bonds, whereas T-bills are considered separately. Dated securities issued at market related rates comprise a large share of internal debt.

Source: Untitled-1 (dea.gov.in)

11. Consider the following statements:

  • Pursuant to the report of H.N. Sanyal Committee, the Contempt of Courts Act, 1971 was passed.
  • The Constitution of India empowers the Supreme Court and the High Courts to punish for contempt of themselves.
  • The Constitution of India defines Civil Contempt and Criminal Contempt.
  • In India, the Parliament is vested with the powers to make laws on Contempt of Court.

(b) 1, 2 and 4 only

(c) 3 and 4 only

  • Statement 1 is correct:  Contempt of Courts Act, 1971 was passed on the recommendation of H. N. Sanyal Committee.
  • Statement 2 is correct:  Constitution Of India empowers Supreme Court and High Courts to punish for contempt of themselves. High courts have the power to punish for contempt for lower courts under respective jurisdiction.
  • Statement 3 is incorrect:  Constitution of India does not define any type of contempt of court, neither Civil contempt nor criminal contempt.
  • Statement 4 is correct:  In India it is the Parliament who has the power to legislate over Contempt of Court.

Source:  Criminal contempt? Not now, My Lords - Frontline (thehindu.com) ,  LakshmiKant

12. With reference to India, consider the following statements:

  • Government law officers and legal firms are recognized as advocates, but corporate lawyers and patent attorneys are excluded from recognition as advocates.
  • Bar Councils have the power to lay down the rules relating to legal education and recognition of law colleges.
  • Statement 1 is incorrect:  Government law officers, legal firms, corporate lawyers and patent attorneys all are recognised as advocates.
  •   Statement 2 is correct:  Bar councils have the powers to lay down rules relating to legal education and recognition of law colleges.

Source: Functions of Bar Council of India - Law Corner

13. Consider the following statements:

  • A bill amending the Constitution requires a prior recommendation of the President of India.
  • When a Constitution Amendment Bill is presented to the President of India, it is obligatory for the President of India to give his/her assent.
  • A Constitution Amendment Bill must be passed by both the Lok Sabha and Rajya Sabha by a special majority and there is no provision for joint sitting.
  • Statement 1 is incorrect:   No prior approval of the President is required for the Constitutional Amendment Act.
  • Statement 2 is correct:   It is obligatory for the President to give his assent, when a Constitutional Amendment Bill is presented before him. The President cannot exercise any veto power regarding the Constitutional Amendment Bill.
  • Statement 3 is correct: Constitutional Amendment Bill needs to be passed by both the houses separately by a special majority, and no joint sitting is allowed regarding Constitutional Amendment Bill.

Source:   LakshmiKan

14. Consider the following statements:

  • The Constitution of India classifies the ministers into four ranks viz. Cabinet Minister, Minister of State with Independent Charge, Minister of State and Deputy Minister.
  • The total number of ministers in the Union Government, including the Prime Minister, shall not exceed 15 percent of the total number of members in the Lok Sabha.
  • Statement 1 is incorrect:   Constitution of India does not define any categorisation of ministers.
  • Statement 2 is correct:   The total number of ministers including prime minister should not exceed 15% of the total number of members of Lok Sabha.

Source:   LakshmiKant

15. Which of the following is/are the exclusive power(s) of Lok Sabha?

  • To ratify the declaration of Emergency
  • To pass a motion of no-confidence against the Council of Ministers
  • To impeach the President of India

(a) 1 and 2

(c) 1 and 3

  • Statement 1 is incorrect: For the ratification of the proclamation of emergency, a resolution for the same must be passed by special majority by the Both Houses separately.
  • Statement 3 is incorrect : Both the Houses have the role in the impeachment of the President. After the impeachment resolution is passed by a majority of two-thirds of the total membership of a House, it is sent to the other House, which should investigate the charges. If the other House also sustains the charges and passes the impeachment resolution by a majority of two-thirds of the total membership, then the President stands removed from his office.
  • Statement 2 is correct : Article 75 of the Constitution says that the council of ministers shall be collectively responsible to the Lok Sabha.

Source: https://indiankanoon.org/doc/1018568/ , LakshmiKant

16. With reference to anti-defection law in India, consider the following statements:

  • The law specifies that a nominated legislator cannot join any political party within six months of being appointed to the House.
  • The law does not provide any time-frame within which the presiding officer has to decide a defection case.

Statement 1 is incorrect: A nominated member of a House becomes disqualified for being a member of the House if he joins any political party after the expiry of six months from the date on which he takes his seat in the House. This means that he may join any political party within six months of taking his seat in the House without inviting this disqualification.  ( Source: LakshmiKant )

Statement 2 is correct: The law does not specify a time-period for the Presiding Officer to decide on a disqualification plea. 

(Source: https://prsindia.org/theprsblog/the-anti-defection-law-explained)

Source: ( Source: LakshmiKant , https://prsindia.org/theprsblog/the-anti-defection-law-explained

17. Consider the following statements:

  • Attorney General of India and Solicitor General of India are the only officers of the Government who are allowed to participate in the meetings of the Parliament of India.
  • According to the Constitution of India, the Attorney General of India submits his resignation when the Government which appointed him resigns.
  • Statement 1 is Incorrect: Solicitor General does not participate in the meetings of Parliament. 
  • Statement 2 is Incorrect: There is no provision regarding thethe procedure and grounds for the removal of Attorney General mentioned in the Constitution except that he/she holds office during the pleasure of the president. Conventionally (but not mandatorily), he/she resigns when the government (council of ministers) resigns or is replaced, as he is appointed on its advice. 

Source: LakshmiKant

18. With reference to the writs issued by the Courts in India, consider the following statements:

  • Mandamus will not lie against a private organization unless it is entrusted with a public duty.
  • Mandamus will not lie against a Company even though it may be a Government Company.
  • Any public minded person can be a petitioner to move the Court to obtain the writ of Quo Warranto.

Mandamus  

It literally means ‘we command’. It is a command issued by the court to a public official asking him to perform his official duties that he has failed or refused to perform. It can also be issued against any public body, a corporation , an inferior court, a tribunal or government for the same purpose. The writ of mandamus cannot be issued (a) against a private individual or body; (b) to enforce departmental instruction that does not possess statutory force; (c) when the duty is discretionary and not mandatory; (d) to enforce a contractual obligation; (e) against the president of India or the state governors; and (f) against the chief justice of a high court acting in judicial capacity.

Quo-Warranto

Quo-Warranto In the literal sense, it means ‘by what authority or warrant’. It is issued by the court to enquire into the legality of claim of a person to a public office. Hence, it prevents illegal usurpation of public office by a person. The writ can be issued only in case of a substantive public office of a permanent character created by a statute or by the Constitution. It cannot be issued in cases of ministerial office or private office . Unlike the other four writs, this can be sought by any interested person and not necessarily by the aggrieved person.

Source:  ( Source: LakshmiKant )

Subject: Current Affairs

19. With reference to Ayushman Bharat Digital Mission, consider the following statements:

  • Private and public hospitals must adopt it.
  • As it aims to achieve universal health coverage, every citizen of India should be part of it ultimately.
  • It has seamless portability across the country.
  • Statement 1 is incorrect: Participation in ABDM is voluntary including for citizens. Participation of a healthcare facility or an institution is also voluntary and shall be taken by the respective management (government or private management). However, once the management decides to register the respective healthcare facility/institution in ABDM, it is essential for all the healthcare professionals serving the said facility/institution to register in Healthcare Professionals Registry so that the institution can become fully integrated with the National Digital Health Ecosystem (NDHE).
  • By elimination, Option (b) is correct

(Source: https://ndhm.gov.in/faq)

20. With reference to Deputy Speaker of Lok Sabha, consider the following statements:

  • As per the Rules of Procedure and Conduct of Business in Lok Sabha, the election of Deputy Speaker shall be held on such date as the Speaker may fix.
  • There is a mandatory provision that the election of a candidate as Deputy Speaker of Lok Sabha shall be from either the principal opposition party or the ruling party.
  • The Deputy Speaker has the same power as of the Speaker when presiding over the sitting of the House and no appeal lies against his rulings.
  • The well established parliamentary practice regarding the appointment of Deputy Speaker is that the motion is moved by the Speaker and duly seconded by the Prime Minister.

(a) 1 and 3 only

(b) 1, 2 and 3

(d) 2 and 4 only

Deputy Speaker of Lok Sabha 

  • Like the Speaker, the Deputy Speaker is also elected by the Lok Sabha itself from amongst its members. He is elected after the election of the Speaker has taken place. The date of election of the Deputy Speaker is fixed by the Speaker. Whenever the office of the Deputy Speaker falls vacant, the Lok Sabha elects another member to fill the vacancy.
  • Upto the 10th Lok Sabha, both the Speaker and the Deputy Speaker were usually from the ruling party. Since the 11th Lok Sabha, there has been a consensus that the Speaker comes from the ruling party (or ruling alliance) and the post of Deputy Speaker goes to the main opposition party.
  • The Deputy Speaker performs the duties of the Speaker’s office when it is vacant. He also acts as the Speaker when the latter is absent from the sitting of the House. In both cases, he assumes all the powers of the Speaker. He also presides over the joint sitting of both the Houses of Parliament, in case the Speaker is absent from such a sitting.
  • At any time before noon on the day preceding the date so fixed, any member may give notice in writing, addressed to the Secretary-General, of a motion that another member be chosen as the Deputy Speaker of the House and the notice shall be seconded by a third member and shall be accompanied by a statement by the member whose name is proposed in the notice that the member proposed is willing to serve as Deputy Speaker 

Source: http://loksabhaph.nic.in/rules/rules.pdf , LakshmiKant

general studies manual paper 1 2022 pdf

21. Among the following crops, which one is the most important anthropogenic source of both methane and nitrous oxide ?

(c) Sugarcane

Methane and nitrous oxide are emitted from conventional and modified rice cultivation systems.

Source: https://www.sciencedirect.com/science/article/pii/S0167880917304607

22. “System of Rice Intensification” of cultivation, in which alternate wetting and drying of rice fields is practised, results in :

  • Reduced seed requirement
  • Reduced methane production
  • Reduced electricity consumption

Select the correct answer using the code given below :

System of Rice Intensification (SRI) includes a method called Alternate Wetting and Drying (AWD) which is a form of controlled or intermittent irrigation of the rice crops.  The benefits from the programme include reduction in methane emissions from traditional rice cultivation, and significant reduction in water consumption. The crop yields are maintained and not affected negatively, and the programme leads to the creation of community organisation and strengthening of networks through the formation of farmer groups. Typically, farmers use diesel to pump large amounts of water to flood their fields. With intermittent flooding, water pumps are run for shorter periods of time, reducing fuel consumption.

Benefits and Impacts of SRI 

  • To increase paddy yields usually by 20-50% and sometimes 100% or more; 
  • To reduce required seeds for transplanting by 60-80%; 
  • To reduce use of chemical fertilizers and agrichemicals; 
  • To reduce irrigation water by 25-50%; 
  • To reduce production costs usually by 10-20%; and
  •  With increased output and reduced costs, farmers’ net income is increased.
  • nvadvisory.com/our-projects/alternate-wetting-drying/#:~:text=System%20of%20Rice%20Intensification%20(SRI,flooding%2C%20reducing%20greenhouse%20gas%20emissions.
  • http://sri.ciifad.cornell.edu/countries/japan/extmats/JSRI_Guideline0312.pdf

23. Which one of the following lakes of West Africa has become dry and turned into a desert ?

(a) Lake Victoria

(b) Lake Faguibine

(c) Lake Oguta

(d) Lake Volta

One example of increasing aridity in Mali appears in Lake Faguibine. These false-color Landsat satellite images of the lake show how it changed over the decades.

Lying at the end of a series of basins watered by the Niger River when it floods, Lake Faguibine has experienced widely fluctuating water levels since the turn of the twentieth century but, at its fullest, has ranked among the largest lakes in West Africa. In 1974, this lake covered roughly 590 square kilometers (230 square miles). Starting in the late 1980s, a drop in precipitation steadily dried the lake. By the late 1990s, the traditional livelihoods of fishing, agriculture, and livestock herding became impractical. Even though normal rainfall resumed after the year 2000, the lake remained nearly dry.

Source: 

https://earthobservatory.nasa.gov/images/8991/drying-of-lake-faguibine-mali

https://www.africanews.com/2021/10/27/climate-change-clears-mali-s-lake-faguibine-displaces-population//

24. Gandikota canyon of South India was created by which one of the following rivers ?

(a) Cauvery

(b) Manjira

(d) Tungabhadra

  • Gandikota is a small village in the Kadapa district of Andhra Pradesh. The village is majorly known for housing the spectacular gorge which is famously adjudged as the Grand Canyon of India. 
  • The stunning gorge has been created by the waters of the famous river Pennar that streams from the Erramala hills.
  • Penna (also known as Pinakini, Pennar, Penner, Penneru (Telugu), Pennai (Tamil)) is a river of southern India.
  • India has always been a popular tourist destination for its historic forts, stunning palaces and dense jungles.
  • Now, a lesser known gorge in the southern state of Andhra Pradesh is drawing visitors.
  • The gorge is a stunning maze of jagged rocks layered in shades of red. The PennaRiver meanders through the canyon floor as it cuts through the Erramala hills.
  • The area is known as the Grand Canyon of India because of its resemblance to the famous US landmark.
  • It's also home to two ancient temples and a 12th Century fort that sits atop the hills surrounding the gorge.

Source: https://www.bbc.com/news/av/world-asia-india-59731622

25. Consider the following pairs Peak Mountains

  • Namcha Barwa - Garhwal Himalaya
  • Nanda Devi – Kumaon Himalaya
  • Nokrek – Sikkim Himalaya

Which of the pairs given above is/are correctly matched ?

Namcha Barwa is in an isolated part of southeastern Tibet rarely visited by outsiders. It stands inside the Great Bend of the Yarlung Tsangpo River as the river enters its notable gorge across the Himalaya, emerging as the Siang and becoming the Brahmaputra. On other hand the Garhwal Himalayas are mountain ranges located in the Indian state of Uttarakhand

Kumaun Himalayas, west-central section of the Himalayas in northern India, extending 200 miles (320 km) from the Sutlej River east to the Kali River. The range, comprising part of the Siwalik Range in the south and part of the Great Himalayas in the north, lies largely within the state of Uttarakhand, northwest of Nepal. It rises to 25,646 feet (7,817 metres) at Nanda Devi, the range’s highest peak.

The Nokrek Biosphere Reserve is located in the northeast of India on the Tura Range, which forms part of the Meghalaya Plateau (average altitude: 600 metres). 

https://www.researchgate.net/figure/Location-map-of-the-Garhwal-Himalaya_fig1_287647789

https://www.britannica.com/place/Kumaun-Himalayas

https://en.unesco.org/biosphere/aspac/nokrek

26. The term “Levant" often heard in the news roughly corresponds to which of the following regions ?

(a) Region along the eastern Mediterranean shores

(b) Region along North African shores stretching from Egypt to Morocco

(c) Region along Persian Gulf and Horn of Africa

(d) The entire coastal areas of Mediterranean Sean

Levant, the region along the eastern Mediterranean shores, roughly corresponding to modern-day Israel, Jordan, Lebanon, Syria, and certain adjacent areas.

https://www.britannica.com/place/Levant

27. Consider the following countries :

  • Turkmenistan

Which of the above have borders with Afghanistan ?

(a) 1, 2 and 5 only

(b) 1, 2, 3 and 4 only

(c) 3, 4 and 5 only

(d) 1, 2, 3, 4 and 5

Afghanistan is completely landlocked—the nearest coast lies along the Arabian Sea, about 300 miles (480 km) to the south—and, because of both its isolation and its volatile political history, it remains one of the most poorly surveyed areas of the world. It is bounded to the 

  • east and south by Pakistan 
  • to the west by Iran
  •  to the north by the Central Asian states of Turkmenistan, Uzbekistan, and Tajikistan.
  •  It also has a short border with Xinjiang, China, at the end of the long, narrow V?kh?n (Wakhan Corridor), in the extreme northeast. 

https://www.britannica.com/place/Afghanistan

28. With reference to India, consider the following statements :

  • Monazite is a source of rare earths.
  • Monazite contains thorium.
  • Monazite occurs naturally in the entire Indian coastal sands in India.
  • In India, Government bodies only can process or export monazite.

(a) 1, 2 and 3 only

  • Monazite is a mineral mainly containing rare earths and thorium-a prescribed substance to be handled by the Department of Atomic Energy (DAE). Accordingly, Indian Rare Earths Ltd. (IREL) wholly owned by the Govt. of India, under the administrative control of the Dept. of Atomic Energy (DAE) utilises monazite mainly for production of rare earth compounds, and thorium, as needed in the Department of Atomic Energy.
  • In-situ monazite established by AMD so far are present in Odisha, Andhra Pradesh, Tamil Nadu, Kerala, West Bengal, and Jharkhand.

https://pib.gov.in/newsite/PrintRelease.aspx?relid=112033

29. In the northern hemisphere, the longest day of the year normally occurs in the:

(a) First half of the month of June

(b) Second half of the month of June

(c) First half of the month of July

(d) Second half of the month of July

  • Summer solstice: The two moments during the year when the path of the Sun in the sky is farthest north in the Northern Hemisphere (June 20 or 21) or farthest south in the Southern Hemisphere (December 21 or 22).
  • At the summer solstice, the Sun travels the longest path through the sky, and that day therefore has the most daylight. 

30. Consider the following pairs : Wetland / Lake Location

  • Hokera Wetland – Punjab
  • Renuka Wetland – Himachal Pradesh
  • Rudrasagar Lake — Tripura
  • Sasthamkotta Lake – Tamil Nadu

How many pairs given above are correctly matched ?

(a) Only one pair

(b) Only two pairs

(c) Only three pairs

(d) All four pairs

  • Hokera Wetland- Jammu and Kashmir
  • Renuka Wetland- Himachal Pradesh
  • Rudrasagar Lake- Tripura
  • Sasthamkotta Lake- Kerala

31. Consider the following :

  • Aarogya Setu

Which of the above are built on top of open-source digital platforms?

An open source platform is any platform that allows access to its source code to any other users or developers. An open source platform is one aspect of a wide availability of open source products. In contrast with closed source software, which is a type of proprietary software that reserves rights only to authorized individuals, open source software aims to allow equal access to anyone and everyone. Open source products are oftentimes part of the free software movement, and the associated Free Software Foundation founded by Richard Stallman in 1985. Rather than being concerned with the cost of the software, the free software movement, and associated foundation, aim to ensure that software users are free to run software, study the software, modify the software, and share such modifications. However, not all open source platforms are part of the free software movement, but they are almost always part of the open source software movement.

Aarogya Setu , India’s coronavirus ( COVID-19 ) contact-tracing and self-assessment app is now open source . 

DigiLocker is an initiative by the government to offer Indian citizens a free platform to store and access important documents. The platform uses several open source technologies to deliver a mass solution and contributes back to the ever-growing community.

DIKSHA is built using Sunbird ED, a fully functional solution building block available as open-source software under MIT license and is part of Sunbird, a “Made in India, Made for the World” digital public good (DPG).

Co-WIN, which is owned by the Ministry of Health and Family Welfare, was developed using open source software. This type of software allows users to modify the source code and distribute it.

Source: https://tech.hindustantimes. com/tech/news/the-power-of- open-source-technology-and- how-india-can-benefit-from-it- 71633005318343.html

https://www.thehindu.com/sci- tech/technology/aarogya-setu- app-is-now-open-source-what- does-it-mean/article31689459. ece

https://github.com/kgisl/ project-ideas/blob/master/ DigiLocker.md#:~:text= DigiLocker%20is%20an% 20initiative%20by,to%20the% 20ever%2Dgrowing%20community .

https://diksha.gov.in/help/ diksha-oss/#:~:text=DIKSHA% 20is%20built%20using% 20Sunbird,digital%20public% 20good%20(DPG) .

32. With reference to Web 3.0, consider the following statements :

  • Web 3.0 technology enables people to control their own data.
  • In Web 3.0 world, there can be blockchain based social networks.
  • Web 3.0 is operated by users collectively rather than a corporation.
  • Web 3.0 is the next version of the internet, where services will run on blockchain. It is a decentralised internet that runs on a public blockchain, which is also used for cryptocurrency transactions.
  • It will be permissionless and democratic. For instance: Twitter will not be able to censor posts and Facebook will not be able to maintain a database of billions of users that can be potentially used to influence elections.
  • In a Web 3.0 universe, people will control their own data and will be able to move around from social media to email to shopping using a single personalized account, creating a public record on the blockchain of all of that activity.
  • All data will be interconnected in a decentralized way, unlike the current generation of the internet (Web 2.0), where data is mostly stored in centralized repositories. 
  • – Three key features of Web 3.0 are: Ubiquity, Semantic Web, Artificial Intelligence and 3D Graphics.

– Examples of Web 3.0: The most recent example of Web 3.0 are the NFTs or non-fungible tokens.

hence , all the statements are correct.

33. With reference to "Software as a Service (SaaS)”, consider the following statements :

  • SaaS buyers can customise the user interface and can change data fields.
  • SaaS users can access their data through their mobile devices.
  • Outlook, Hotmail and Yahoo! Mail are forms of SaaS. .
  • Software as a service (SaaS) allows users to connect to and use cloud-based apps over the Internet. Common examples are email, calendaring and office tools (such as Microsoft Office 365). 
  • 2. SaaS provides a complete software solution which you purchase on a pay-as-you-go basis from a cloud service provider. You rent the use of an app for your organisation and your users connect to it over the Internet, usually with a web browser. Statement 1 is incorrect.
  • Access app data from anywhere. With data stored in the cloud, users can access their information from any Internet-connected computer or mobile device. And when app data is stored in the cloud, no data is lost if a user’s computer or device fails. Statement 2 is correct.
  • If you have used a web-based email service such as Outlook, Hotmail or Yahoo! Mail, then you have already used a form of SaaS. With these services, you log into your account over the Internet, often from a web browser. The email software is located on the service provider’s network and your messages are stored there as well. You can access your email and stored messages from a web browser on any computer or Internet-connected device. Statement 3 is correct.

Source: https://azure.microsoft.com/ en-in/overview/what-is-saas/

  34. Which one of the following statements best reflects the idea behind the “Fractional Orbital Bombardment System” often talked about in media ?

(a) A hypersonic missile is -launched into space to counter the asteroid approaching the Earth and explode it in space.

(b) A spacecraft lands on another planet after making several orbital motions.

(c) A missile is put into a stable orbit around the Earth and deorbits over a target on the Earth.

(d) A spacecraft moves along a comet with the same speed and places a probe on its surface.'

A Fractional Orbital Bombardment System (FOBS) is a warhead delivery system that uses a low earth orbit towards its target destination. Just before reaching the target, it deorbits through a retrograde engine burn.

The idea behind FOBS is, a warhead is put into a stable orbit and it deorbits over the target.

If the target and the launch position is lined up and the warhead keeps going round, it will complete a circle.Obviously, with the earth moving during this, it is more of a spiral than a ring, but the horizontal movement is countered in such a way that the warhead still goes over the target.

Source: https://www.financialexpress.com/defence/chinas-fractional-orbital-bombardment-system-impact-on-indias-nuclear-deterrence-posture/2356471/

35. Which one of the following is the context in which the term “qubit” is mentioned ?

(a) Cloud Services

(b) Quantum Computing

(c) Visible Light Communication Technologies

(d) Wireless Communication Technologies

The phrase ‘quantum supremacy’ was coined in the year 2011 by John Preskill, Professor of Theoretical Physics at the California Institute of Technology.

  • The concept is related to the speed at which a quantum computer performs.
  • The common digital computing requires the data to be encoded into binary digits (bits), each of which is always in one of two definite states (0 or 1).
  • Quantum computation uses quantum bits (qubits).

Quantum Bit (Qubit)

  • Entanglement means that qubits in a superposition state can be correlated with each other; that is, the state of one qubit (whether it is a 1 or a 0) can depend on the state of another qubit. It means that particles remain connected so that actions performed on one affect the other, even when separated by great distances. The phenomenon so riled Albert Einstein that he called it "spooky action at a distance."
  • Using these two principles, qubits can act as more sophisticated switches, enabling quantum computers to function in ways that allow them to solve difficult problems that are unmanageable using today’s computers.

36. Consider the following communication technologies :

  • Closed-circuit Television
  • Radio Frequency Identification
  • Wireless Local Area Network

Which of the above are considered Short-Range devices/technologies ?

(d). 1, 2 and 3

Short Range Devices (SRD) are radio devices that offer a low risk of interference with other radio services, usually because their transmitted power, and hence their range, is low. The definition 'Short Range Device' may be applied to many different types of wireless equipment, including various forms of:

  • Access control (including door and gate openers)
  • Alarms and movement detectors
  • Closed-circuit television (CCTV)
  • Cordless audio devices, including wireless microphones
  • Industrial control
  • Local Area Networks
  • Medical implants
  • Metering devices
  • Remote control
  • Radio frequency identification (RFID)
  • Road Transport Telematics

Short range devices often benefit from a relaxed regulatory regime compared with other radio communications equipment.  As a general principle, a user is licence free to operate such equipment, some specific cases may require an individual licence.

Source: https://www.etsi.org/ technologies/short-range- devices

37. Consider the following statements :

  • Biofilms can form on medical implants within human tissues.
  • Biofilms can form on food and food processing surfaces.
  • Biofilms can exhibit antibiotic resistance.

Which of the statements given above are correct ?

  • Biofilm is an association of micro-organisms in which microbial cells adhere to each other on living or non-living surfaces within a self-produced matrix of extracellular polymeric substance.
  • Biofilm formation is a multi-step process starting with attachment to a surface then formation of micro-colony that leads to the formation of three dimensional structure and finally ending with maturation followed by detachment.
  • During biofilm formation many species of bacteria are able to communicate with one another through a specific mechanism called quorum sensing. It is a system of stimulus to coordinate different gene expression.
  • Bacterial biofilm is less accessible to antibiotics and the human immune system and thus poses a big threat to public health because of its involvement in a variety of infectious diseases. Statement 3 is correct.
  • Biofilms may form on living or non-living surfaces and can be prevalent in natural, industrial and hospital settings.
  • Biofilms can be present on the teeth of most animals as dental plaque, where they may cause tooth decay and gum disease. Statement 1 is correct
  • A combination of several pathogens can synergistically interact to form biofilms in the food industry. In food-processing environments, bacteria are able to exist as multispecies biofilms, from where both spoilage and pathogenic bacteria can contaminate food. For instance in the fishing industry, fresh fish products can suffer from biofilm formation by mixed pathogenic species (Aeromonashydrophila, L. monocytogenes, S. enterica, or Vibrio spp.), which can imply significant health and economic issues. Synergistic interactions have been observed in a fresh-cut produce processing plant, where E. coli interacted with Burkholderiacaryophylli and Ralstoniainsidiosa to form mixed biofilms. Statement 2 is correct.

Source: https://www.livescience.com/ 57295-biofilms.html

38. Consider the following statements in respect of probiotics :

  • Probiotics are made of both bacteria and yeast.
  • The organisms in probiotics are found in foods we ingest but they do not "naturally occur in our gut.
  • Probiotics help in the digestion of milk sugars.

Which of the statements given above is/are correct ?

(d) 2 and 3

  • Probiotics are foods that are made up of good live bacteria or yeasts that naturally stay in the human body. Human body is a host to good and bad bacteria. Whenever one gets an infection, there is more bacteria that is bad. 

Probiotic supplements add good bacteria to the human body.

  • These are a combination of beneficial bacteria and yeasts that naturally stay in the human body. Bacteria is not always a negative addition to the body, it is positive too. 

Probiotics are part of a larger picture concerning bacteria and your body which is called your microbiome. These microbes are a combination of:

  • Fungi (including yeasts)

Statement 1 is correct.

 Benefits of Probiotics in a Nutshell

  • Help the digestive system of the body
  • Keeping bad bacteria from getting out of control and making one ill
  • Create vitamins in the body
  • Help support the cells that line your gut to prevent bad bacteria that you may have consumed from entering your blood.
  • Breakdown and absorption of medications

Statement 2 is incorrect. And statement 3 is correct.

Source: https://www.healthline.com/ health/probiotics-and- digestive-health#:~:text= Lactobacillus%3A%20This% 20genus%20of%20bacteria,the% 20body's%20absorption%20of% 20minerals .

39. In the context of vaccines manufactured to prevent COVID-19 pandemic, consider the following statements:

  • The Serum Institute of India produced COVID-19 vaccine named Covishield using mRNA platform.
  • Sputnik V vaccine is manufactured using vector based platform.
  • COVAXIN is an inactivated pathogen based vaccine.
  • COVISHIELD vaccine is based on the platform which uses a recombinant, replication-deficient chimpanzee adenovirus vector encoding the SARS-CoV-2 Spike (S) glycoprotein. Following administration, the genetic material of part of coronavirus is expressed which stimulates an immune response . Hence the statement is incorrect.
  • Sputnik V is the world's first registered vaccine based on a well-studied human adenovirus vector platform. It has been approved for use in 71 countries with a total population of 4 billion people. The vaccine is named after the first Soviet space satellite. The launch of Sputnik-1 in 1957 reinvigorated space research around the world, creating a so-called “Sputnik moment” for the global community. The vaccine’s efficacy is 97.6%, based on the analysis of data on the incidence of coronavirus among Russians vaccinated with both vaccine components between December 5, 2020 and March 31, 2021. Statement 2 is correct.
  • Covaxin is an inactivated viral vaccine. This vaccine is developed with Whole-Virion Inactivated Vero Cell-derived technology. They contain inactivated viruses, which cannot infect a person but still can teach the immune system to prepare a defence mechanism against the active virus. Statement 3 is correct.

40. If a major solar storm (solar-flare) reaches the Earth, which of the following are the possible. effects on the Earth ?

  • GPS and navigation systems could fail.
  • Tsunamis could occur at equatorial regions.
  • Power grids could be damaged.
  • Intense auroras could occur over much of the Earth.
  • Forest fires could take place over much of the planet.
  • Orbits of the satellites could be disturbed.
  • Shortwave radio communication of the aircraft flying over polar regions could be interrupted.

(a) 1, 2, 4 and 5 only

(b) 2, 3, 5, 6 and 7 only

(c) 1,3, 4, 6 and 7 only

(d) 1, 2, 3, 4, 5, 6 and 7.

Impact of Solar Flares and CMEs on Earth

  • Space-dependent services: Solar storms can hit operations of space-dependent services like global positioning systems (GPS), radio, and satellite communications.
  • Radio communication: Geomagnetic storms interfere with high-frequency radio communications and GPS navigation systems.
  • Magnetosphere: CMEs, with ejectiles loaded with matter travelling at millions of miles an hour, can potentially create disturbances in the magnetosphere, the protective shield surrounding the Earth.
  • Astronauts: Astronauts on spacewalks face health risks from possible exposure to solar radiation outside the Earth’s protective atmosphere.
  • Other: Aircraft flights, power grids, and space exploration programmes are vulnerable.

Source: https://www.space.com/solar- flares-effects-classification- formation

41. "Climate Action Tracker" which monitors the emission reduction pledges of different countries is a :

(a) Database created by coalition of research organisations

(b) Wing of "International Panel of Climate Change"

(c) Committee under "United Nations Framework Convention on Climate Change"

(d) Agency promoted and financed by United Nations Environment Programme and World Bank

The Climate Action Tracker is an independent scientific analysis that tracks government climate action and measures it against the globally agreed Paris Agreement aim of "holding warming well below 2°C, and pursuing efforts to limit warming to 1.5°C." A collaboration of two organisations, Climate Analytics and NewClimate Institute, the CAT has been providing this independent analysis to policymakers since 2009. CAT quantifies and evaluates climate change mitigation targets, policies and action. It also aggregates country action to the global level, determining likely temperature increases during the 21st century using the MAGICC climate model. CAT further develops sectoral analysis to illustrate required pathways for meeting the global temperature goals.

Source: https://climateactiontracker.org/about/

42. Consider the following statements :

  • "The Climate Group" is an international non-profit organization that drives climate action by building large networks and runs them.
  • The International Energy Agency in partnership with the Climate Group launched a global initiative "EP100".
  • EP100 brings together leading companies committed to driving innovation in energy efficiency and increasing competitiveness while delivering on emmission reduction goals.
  • Some Indian companies are members of EP100.
  • The International Energy Agency is the Secretariat to the "Under2 Coalition".

(a) 1, 2, 4 and 5

(b) 1, 3 and 4 only

(c) 2, 3 and 5 only

  • CLIMATE GROUP IS an international non-profit founded in 2003, with offices in London, New York, New Delhi, Amsterdam and Beijing. Their mission is to drive climate action. Fast.   This group builds and run networks. Statement 1 is correct.
  •   EP100 is a global initiative led by the international non-profit Climate Group, bringing together over 120 energy smart businesses committed to measuring and reporting on energy efficiency improvements. Statement 2 is incorrect. Statement 3 is correct.
  • Mahindra and Mahindra became the first Indian company to join the EP100 initiative. Statement 4 is correct.
  • Climate Group is the Secretariat to the Under2 Coalition and works with governments to accelerate climate action through four work streams:

Policy action

Transparency

Diplomacy 

 Statement 5 is incorrect.

Source: https://www.theclimategroup. org/about-us

https://www.theclimategroup. org/about-ep100

https://www.theclimategroup. org/under2-coalition#:~:text= Climate%20Group%20is%20the% 20Secretariat,Transparency

43. "If rainforests and tropical forests are the lungs of the Earth, then surely wetlands function as its kidneys." Which one of the following functions of wetlands best reflects the above statements?

(a). The water cycle in wetlands involves surface runoff, subsoil percolation and evaporation.

(b). Algae from the nutrient base upon which fish, crustaceans, molluscs, birds, reptiles and mammals thrive.

(c). Wetlands play a vital role in maintaining sedimentation balance and soil stabilization.

(d). Aquatic plants absorb heavy metals and excess nutrients.

* Correct Option: (a/d) Explanation:

The water cycle in wetlands involves surface runoff, subsoil percolation and evaporation.

Explanation: Your kidneys remove wastes and extra fluid from your body. Your kidneys also remove acid that is produced by the cells of your body and maintain a healthy balance of water, salts, and minerals. In the same manner As sediment, excess nutrients and chemicals flow off of the land, wetlands filter the run off before it reaches open water. Nutrients are stored and absorbed by plants or microorganisms. Sediment settles at the bottom after reaching an area with slow water flow

Source: sciencing.com

44. In the context of In WHO Air Quality Guidelines, consider the following statements:

  • The 24-hour mean of PM2.5 should not exceed 15 ug/m3 and annual mean of PM2.5 should not exceed 5 µg/m³.
  • In a year, the highest levels of ozone pollution occur during the periods of inclement weather.
  • PM10 can penetrate the lung barrier and enter the bloodstream.
  • Excessive ozone in the air can trigger asthma.

(a) 1, 3 and 4

(b) 1 and 4 only

(c) 2, 3 and 4

(d) 1 and 2 only

  • The updated guidelines state that annual average concentrations of PM 2.5  should not exceed 5 µg/m 3 , while 24-hour average exposures should not exceed 15 µg/m 3
  • The highest levels of ozone pollution occur during periods of sunny weather.
  • PM10 are capable of penetrating deep into the lungs but PM2.5 can even enter the bloodstream        
  • Long-term exposure to ozone is linked to aggravation of asthma, and is likely to be one of many causes of asthma development. Studies in locations with elevated concentrations also report associations of ozone with deaths from respiratory causes.                      

Source:   https://www.c40knowledgehub.org/s/article/WHO-Air-Quality-Guidelines?language=en_US

https://www.who.int/news-room/fact-sheets/detail/ambient-(outdoor)-air-quality-and-health

https://www.who.int/news/item/22-09-2021-new-who-global-air-quality-guidelines-aim-to-save-millions-of-lives-from-air-pollution                 

https://www.epa.gov/ground-level-ozone-pollution/health-effects-ozone-pollution#:~:text=Long%2Dterm%20exposure%20to%20ozone,with%20deaths%20from%20respiratory%20causes .

45. With reference to "Gucchi" sometimes mentioned in the news, consider the following statements:

  • It is a fungus.
  • It grows in some Himalayan forest areas.
  • It is commercially cultivated in the Himalayan foothills of north- eastern India.

(c) 1 and 2

  • Guchhi mushroom is a species of fungus in the family Morchellaceae of the Ascomycota.
  • Grows in himalayan foothills
  • The mushrooms cannot be cultivated commercially and grow in conifer forests across temperature regions, and the foothills in Himachal Pradesh, Uttaranchal, and Jammu and Kashmir. 

Source:   https://indianexpress.com/article/lifestyle/food-wine/gucchi-mushrooms-cost-health-benefits-6484874/

46. With reference to polyethylene terephthalate, the use of which is so widespread in our daily lives, consider the following statements :

  • Its fibres can be blended with wool and cotton fibres to reinforce their properties.
  • Containers made of it can be used to store any alcoholic beverage.
  • Bottles made of it can be recycled into other products.
  • Articles made of it can be easily disposed of by incineration without causing greenhouse gas emmisions.

(a) 1 and 3

(b) 2 and 4

(c) 1 and 4

  • They are often used in durable-press blends with other fibres such as rayon, wool, and cotton, reinforcing the inherent properties of those fibres while contributing to the ability
  • PET is completely recyclable, and is the most recycled plastic in the U.S and worldwide. More than 1.5 billion pounds of used PET bottles and containers are recovered in the United States each year for recycling.
  • The burning of plastics releases toxic gases like dioxins, furans, mercury and polychlorinated biphenyls (better known as BCPs) into the atmosphere, and poses a threat to vegetation, and human and animal health.
  • Liquor becomes carcinogenic when stored in plastic bottles.

47. Which of the following is not a bird ?

(a) Golden Mahseer

(b) Indian Nightjar

(c) Spoonbill

(d) White Ibis

  • Golden Mahseer —a large freshwater fish also called the tiger of the water and found only in the Cauvery river basin 
  • The Indian nightjar species are nocturnal birds with long pointed wings and short legs
  • The Spoonbill is a unique aquatic bird with an odd-shaped, flattened beak
  • White Ibises are   large wading birds with football-shaped bodies

Source:   https://www.thehindu.com/sci-tech/energy-and-environment/the-hump-backed-mahseer-critically-endangered/article26653559.ece

https://indianbirds.thedynamicnature.com/2017/02/indian-nightjar-caprimulgus-asiaticus.html

https://animals.net/spoonbill/

https://www.allaboutbirds.org/guide/White_Ibis/id#:~:text=White%20Ibises%20are%20large%20wading,hold%20out%20straight%20in%20flight .

48. Which of the following are nitrogen-fixing plants ?

  • Purslane (Kulfa)

(a) 1, 3 and 4 only

(b) 1, 3, 5 and 6 only

(c) 2, 4, 5 and 6 only

(d) 1, 2, 4,5 and 6

Amaranth, spinach, purslane are not nitrogen fixing plants and we have to provide nitrogen through fertilizers for their proper growth and development.

https://eos.com/blog/nitrogen- fixation/

https://www.thehindu.com/ features/homes-and-gardens/ gardens/natural-nutrition-for- plants/article5735861.ece

49. "Biorock technology" is talked about in which one of the following situations ?

(a) Restoration of damaged coral reefs

(b) Development of building materials using plant residues

(c) Identification of areas for exploration/extraction of shale gas

(d) Providing salt licks for wild animals in forests/protected areas.

Explanation: BIOROCK OR Mineral Accretion Technology is a coral reef restoration technology that utilizes low voltage electricity to improve the health and growth rates of corals and other marine organisms. 

  • The technology works by passing a small amount of electrical current through electrodes in the water.
  • When a positively charged anode and negatively charged cathode are placed on the seafloor, with an electric current flowing between them, calcium ions combine with carbonate ions and adhere to the structure (cathode).
  • This results in calcium carbonate formation. Coral larvae adhere to the CaCO3 and grow quickly.
  • Fragments of broken corals are tied to the biorock structure, where they are able to grow at least four to six times faster than their actual growth as they need not spend their energy in building their own calcium carbonate skeletons.

Source: https://www.thehindu.com/sci-tech/science/india-begins-coral-restoration-in-gulf-of-kachchh/article30645770.ece

50. The “Miyawaki method” is well known for the:

(a) Promotion of commercial farming in arid and semi-arid areas

(b) Development of gardens using genetically modified flora

(c) Creation of mini forests in urban areas

(d) Harvesting wind energy on coastal areas and on sea surfaces

  • The method involves planting two to four trees per square metre. Miyawaki forests grow in two to three years and are self-sustaining.

Source: https://urban-forests.com/miyawaki-method/

51. In the Government of India Act 1919, the functions of Provincial Government were divided into “Reserved” and “Transferred” subjects. Which of the following were treated as “Reserved” subjects?

  • Administration of Justice
  • Local Self-Government
  • Land Revenue

(a) 1, 2 and 3

(b) 2, 3 and 4

(c) 1, 3 and 4

(d) 1, 2 and 4

  • The reserved subjects came under the heading of law and order and included justice , the police, land revenue, and irrigation. The transferred subjects (i.e., those under the control of Indian ministers) included local self-government, education, public health , public works , and agriculture, forests, and fisheries. 

Source: https://www.britannica.com/topic/dyarchy

52. In medieval India, the term “Fanam” referred to:

(a) Clothing

(c) Ornaments

(d) Weapons

  • Fanam.—Anglicised form of Tamil pan am (q. v.); Sanskrit pana (q. v.); a gold coin equal to (1/20) of a vara ha.
  • Note: fanam is defined in the “Indian epigraphical glossary” as it can be found on ancient inscriptions commonly written in Sanskrit, Prakrit or Dravidian languages

Source: https://www.wisdomlib.org/definition/fanam

53. Consider the following freedom fighters:

1. Barindra Kumar Ghosh

2. Jogesh Chandra Chatterjee

3. Rash Behari Bose

Who of the above was/were actively associated with the Ghadar Party?

  • Barindra Kumar Ghosh- AnushilanSamiti
  • Jogesh Chandra Chatterjee- AnushilanSamiti
  • Rash Behari Bose- Ghadr Party

Source: https://www.livehistoryindia.com/story/people/ghadar-party-call-for-freedom-across-oceans

54. With reference to the proposals of Cripps Mission, consider the following statements:

  • The Constituent Assembly would have members nominated by the Provincial Assemblies as well as the Princely States.
  • Any Province, which is not prepared to accept the new Constitution would have the right to sign a separate agreement with Britain regarding its future status.
  • A Constituent Assembly would consist of the members elected by the Lower House of the Indian Legislature and the representatives of the Princely States nominated by their rulers.

https://www.knowledgeboat.com/question/mention-any-three-proposals-of-the-cripps-mission--247215673319071800

55. With reference to Indian history, consider the following texts:

  • Nettipakarana
  • Parishishtaparvan
  • Avadanashataka
  • Trishashtilakshana Mahapurana

Which of the above are Jaina texts?

(b) 2 and 4 only

(d) 2, 3 and 4

  • The Avadanashataka or "Century of Noble Deeds" is an anthology in Sanskrit of one hundred Buddhist legends, approximately dating to the same time as the Ashokavadana.
  • Option 3 is incorrect. So by eliminating, we get option (b) as correct.
  • The Parishishtaparvan also known as the Sthaviravalicharitra is a 12th-century Sanskrit mahakavya by Hemachandra which details the histories of the earliest Jain teachers.
  • Mahapurana or Trishashthilkshana Mahapurana is a major Jain text composed largely by "Acharya'' Jinasena during the rule of Rashtrakuta ruler Amoghavarsha and completed by his pupil Gunabhadra in the 9th century CE. Mahapurana consists of two parts. The first part is Adi purana written by "Acharya'' Jinasena. The second part is Uttarapurana which is the section composed by Gunabhadra.

https://synonymsbot.com/gunabhadra

https://hyperleap.com/topic/Parishishtaparvan

56. With reference to Indian History, consider the following pairs:

Historical person Known as

  • Aryadeva — Jaina scholar
  • Dignaga — Buddhist scholar
  • Nathamuni — Vaishnava scholar

How many pairs given above are correctly matched?

(a) None of the pairs

(b) Only one pair

(c) Only two pairs

(d) All three pairs

  • Aryadeva (3rd century), a disciple of Nagarjuna, is a central figure in the development of early Indian Madhyamaka philosophy. Aryadeva’s Hundred Verses Treatise ( Bai lun ) was one of the three basic texts of the Chinese Madhyamaka school founded by the central Asian monk Kumarajiva (b. 344–d. 413), which accordingly was called the Sanlun (Jpn. Sanron), or “three-treatise” school. According to the biography that Kumarajiva translated into Chinese, Aryadeva was born into a South Indian Brahmin family, became Nagarjuna’s disciple, was renowned for his skill in debate, and was murdered by a student of a defeated teacher
  • Dignaga, (born c. 480 CE—died c. 540), Buddhist logician and author of the Praman asamuccaya (“Compendium of the Means of True Knowledge”), a work that laid the foundations of Buddhist logic.
  • Sri Ranganathamuni, popularly known as Sriman Nathamuni, was a Vaishnava theologian who collected and compiled the Nalayira Divya Prabandham. 

Source: https://www.thehindu.com/society/faith/nathamuni-and-alavandar/article34038058.ece

57. With reference to Indian history, consider the following statements:

  • The first Mongol invasion of India happened during the reign of Jalal-ud-din Khalji.
  • During the reign of Ala-ud-din Khalji, one Mongol assault marched up to Delhi and besieged the city.
  • Muhammad-bin-Tughlaq temporarily lost portions of north-west of his kingdom to Mongols.
  • Statement 1 is incorrect: Chengez Khan invaded India during the reign of Iltumish for the first time. He was the founder and first Great Khan of the Mongol Empire.
  • Statement 2 is correct: During the reign of Ala-ud-din Khalji, one of the Mongol invasion reached till the outskirts of Delhi city and besieged it.
  • Statement 3 is incorrect: Muhammad-bin-Tughlaq defeated the Mongols and he had not lost any of the portions of his kingdom to them.
  • Source: The Mongols and Delhi Sultanate- Cambridge

58. With reference to Indian history, who of the following were known as “Kulah-Daran”?

(a) Arab merchants

(b) Qalandars

(c) Persian calligraphists

(d) Sayyids

  • The Sayyids were claimed to descent from the Prophet through his daughter Fatima. They Commanded special respect in Muslim society.
  • Even the Timur protected the life of Sayyids during his invasion in India. Although his policy was one of general slaughter.
  • The sayyids put on a pointed cap (kulah) and they were known as ‘Kulah Daran’ during Delhi sultanate.
  • Source: History of Medieval India by V.D Mahajan

59. With reference to Indian history, consider the following statements:

  • The Dutch established their factories/werehouses on the east coast on lands granted to them by Gajapati rulers.
  • Alfonso de Albuquerque captured Goa from the Bijapur Sultanate.
  • The English East India Company established a factory at Madras on a plot of land leased from a representative of the Vijayanagara empire.
  • Statement 1 is incorrect: Next to the Portuguese, the Dutch set their feet in India.
  • In 1602, the United East India Company of the Netherlands was formed and given permission by the Dutch government to trade in the East Indies including India.
  • Gajapati Rule has declined in 1541. The last ruler was Kakharua Deva.
  • Statement 2 is correct: Alfanso de Albuquerque captured Goa from the Bijapur sultanate king Adil Shahis with the help of Vijaynagara Empire.
  • Statement 3 is correct: the English East India Company established a factory in Madras in 1639 on land leased from representatives of Vijayanagara Empire called the Nayakas.
  • Source: The Hindu newspaper- founders of Madras city, EAST INDIA COMPANY FACTORY RECORDS from British Library, London.

60. According to Kautiyla’s Arthahastra, which of the following are correct?

  • A person could be a slave as a result of a judicial punishment.
  • If a female slave bore her master a son, she was legally free.
  • If a son born to a female slave was fathered by her master, the son was entitled to the legal status of the master’s son.

Rules for slavery as per the Arthshastra

  • A person enslaved because of the judicial punishment has been mentioned as dandpraneet u). This kind of slavery was of a fixed period because a Dandapraneet could be freed by serving his sentence. They were the criminals in the prison. They had to serve like slaves but at the same time Kautilya provides that they could be released after their period of punishment was completed.
  • THE selling or mortgaging by kinsmen of the life of a Súdra who is not a born slave, and has not attained majority, but is an Arya in birth shall be punished with a fine of 12 panas; of a Vaisya, 24 panas; of a Kshatriya, 36 panas; and of a Bráhman, 48 panas. If persons other than kinsmen do the same, they shall be liable to the three amercements and capital punishment respectively: purchasers and abettors shall likewise be punished. It is no crime for Mlechchhas to sell or mortgage the life of their own offspring. But never shall an Arya be subjected to slavery.
  • If a slave who is less than eight years old and has no relatives, no matter whether he is born a slave in his master’s house, or fallen to his master’s share of inheritance, or has been purchased or obtained by his master in any other way, is employed in mean avocations against his will or is sold or mortgaged in a foreign land;[9] or if a pregnant female slave is sold or pledged without any provision for her confinement, her master shall be punished with the first amercement. The purchaser and abettors shall likewise be punished.
  • When a child is begotten on a female slave by her master, both the child and its mother shall at once be recognised as free.[10] If, for the sake of subsistence, the mother has to remain in her bondage, her brother and sister shall be liberated.
  • Slaves had been treated as Chattels and they could be inherited from a father to his son, and slaves received in such a manner are described as Dayah-agatah in the Arthasastra of Kautilya. Dayad means given and this would suggest that slaves were given from father.

https://archive.org/stream/SlaveryInTheMauryanPeriod-300BC-200BC-RekhaRaniSharma/Slavery%20in%20the%20Mauryan%20Period%20c.%20300%20B.C.%20-%20c.%20200%20B.C.%20Rekha%20Rani%20Sharma_djvu.txt  

https://www.wisdomlib.org/hinduism/book/kautilya-arthashastra/d/doc366096.html

61. Consider the following statements :

  • Tight monetary policy of US Federal Reserve could lead to capital flight.
  • Capital flight may increase the interest cost of firms with existing External Commercial Borrowings (ECBs).
  • Devaluation of domestic currency decreases the currency risk associated with ECBs.
  • Statement 1 is correct: Tight monetary policy of the US federal Reserve could lead to capital flight. It is also called ‘Taper Tantrum’.
  • Statement 2 is correct: Interest risk associated with capital flight: capital flight increases the interest rate on firms with Existing borrowings.
  • Statement 3 is incorrect: Capital Risk associated with capital flight: Devaluation of domestic currency will increase the risk with external commercial borrowings.
  • It will increase the liability of domestic firms in terms of domestic currency.

Source: Press release of January 2022, RBI

62. Consider the following States :

  • Andhra Pradesh
  • Himachal Pradesh

How many of the above are generally known as tea-producing States ?

(a) Only one State

(b) Only two States

(c) Only three States

(d) All four States

  • Option (d) is correct: Assam, West Bengal, Tamil Nadu, and Kerala are the major tea producing states in India.
  • Other states that produce tea include Himachal Pradesh, Uttarakhand, Meghalaya, Andhra Pradesh and Tripura.

Source: Tea association of India, News- Tea estates facing Crisis (Economic Times)

63. Consider the following statements :

  • In India, credit rating agencies are regulated by Reserve Bank of India.
  • The rating agency popularly known as ICRA is a public limited company.
  • Brickwork Ratings is an Indian credit rating agency.
  • Statement 1 is incorrect:  Credit Rating Agencies (CRA) analyze a debtor’s ability to repay the debt and also rate their credit risk.
  • All the credit rating agencies in India are regulated by SEBI (Credit Rating Agencies) Regulations, 1999 of the Securities and Exchange Board of India Act, 1992.
  • There are a total of six credit agencies in India viz, CRISIL, CARE, ICRA, SMREA, Brickwork Rating, and India Rating and Research Pvt. Ltd.
  • Statement 2 is correct: ICRA Limited is a public limited company that was set up in 1991 in Gurugram. The company was formerly known as Investment Information and Credit Rating Agency of India Limited.
  • Statement 3 is correct: Brickwork Ratings is recognised as external credit assessment agency (ECAI) by Reserve Bank of India (RBI) to carry out credit ratings in India.
  • Brickwork Rating was established in 2007 and is promoted by Canara Bank. It offers ratings for bank loans, SMEs, corporate governance rating, municipal corporation, capital market instrument, and financial institutions.

64. With reference to the ‘Banks Board Bureau (BBB), which of the following statements are correct ?

  • The Governor of RBI is the Chairman of BBB.
  • BBB recommends for the selection of heads for Public Sector Banks.
  • BBB helps the Public Sector Banks in developing strategies and capital raising plans.
  • Statement 1 is incorrect : It is not mandated that the Governor of RBI will be the chairman of Bank Board Bureau (BBB).
  • Statements 2 & 3 are correct: The BBB is empowered to select the heads of Public sector Banks. It also helps PSBs to develop strategies and capital raising plans.
  • The broad agenda of the Banks Board Bureau was the administration of state-owned lenders. Its functions involve:
  • providing assistance to Public Sector Banks to restructure their business strategies
  • Assisting banks with the strategies to deal with issues of bad loans or stressed assets
  • Strategies for raising capitals through innovative financial instruments and methods
  • Recommendations to the government on top-level appointments like full-time Directors, non-Executive Chairman in PSBs.
  • suggest plans for consolidation and merger with other banks while they are trapped in the problem of high collective gross NPAs.
  • To advise the Central Government on matters relating to appointments, confirmation or extension of tenure and termination of services of the Directors of nationalised banks.
  • To build a data bank containing data relating to the performance of nationalised banks and its officers.
  • To advise the Central Government on the formulation and enforcement of a code of conduct and ethics for managerial personnel in nationalised banks.
  • To advise the Central Government on evolving suitable training and development programs for managerial personnel in nationalised banks.
  • So By Elimination the other two statements are correct.

Source: Bank Board Bureau official website, Indian Express

65. With reference to Convertible Bonds, consider the following statements :

  • As there is an option to exchange the bond for equity, Convertible Bonds pay a lower rate of interest.
  • The option to convert to equity affords the bondholder a degree of indexation to rising consumer prices.
  • Both the statements are correct.
  • Convertible bonds: Convertible bonds are hybrid securities that offer investors the best of both stocks and bonds. Like any other kind of bond, they provide a guaranteed income stream and pay back the amount you originally lent the company.
  • Convertible bonds typically carry lower interest rates payments than straight corporate bonds—the savings in interest expense can be significant. Investors accept the lower interest payments because the conversion option offers the opportunity to benefit from increases in the stock price. Hence statement 1 is correct.
  • Governments might use indexation as a way to potentially alleviate the negative effects inflation can have on the recipients of transfer payments and entitlements. Social Security payments, for example, are indexed to the annual increase in the Consumer Price Index.

Source: Corporate Finance Institute paper on Convertible Bonds

https://www.forbes.com/advisor/investing/convertible-bonds/

66. Consider the following:

  • Asian Infrastructure Investment Bank
  • Missile Technology Control Regime
  • Shanghai Cooperation Organisation

India is a member of which of the above ?

(c) 2 and 3 only

  • I ndia is the member of all the Organizations.
  • The SCO currently comprises eight Member States (China, India, Kazakhstan, Kyrgyzstan, Russia, Pakistan, Tajikistan and Uzbekistan).
  • Missile Technology Control Regime: On 27 June, India became a full member of the MTCR after a “deal” was struck with Italy. This marks the first entry into any multilateral export control regime for India.
  • Asian Infrastructure Investment Bank: On 24 October 2014, twenty-one countries signed a Memorandum of Understanding (MOU) regarding the AIIB in Beijing, China: Bangladesh, Brunei, Cambodia, India, Kazakhstan, Kuwait, Laos, Malaysia, Myanmar, Mongolia, Nepal, Oman, Pakistan, Philippines, Qatar, Singapore, Sri Lanka, Thailand, Uzbekistan and Vietnam.

Source: Organizations in News, The Hindu

67. Consider the following statements :

  • Vietnam has been one of the fastest growing economies in the world in the recent years.
  • Vietnam is led by a multi-party political system.
  • Vietnam's economic growth is linked to its integration with global supply chains and focus on exports.
  • For a long time Vietnam's low labour costs and stable exchange rates have attracted global manufacturers.
  • Vietnam has the most productive e-service sector in the Indo-Pacific region.

(a) 2 and 4

(b) 3 and 5

(d) 1 and 2

  • Statement 1 is correct: According to a forecast by PricewaterhouseCoopers in February 2017, Vietnam may be the fastest-growing of the world's economies, with a potential annual GDP growth rate of about 5.1%, which would make its economy the 10th-largest in the world by 2050.

Statement 2 is incorrect: Vietnam is a Marxist–Leninist one-party state based on democratic centralism.

  • Statement 3 is correct: Vietnam’s economic growth is linked to its integration with global supply chains and focus on exports.
  • Statement 4 is correct: Vietnam has low labour costs and stable exchange rates which attract global investments.

Source: World Bank news update https://www.worldbank.org/en/news/press-release/2022/01/13/vietnam-s-economic-growth-is-expected-to-accelerate-to-5-5-in-2022-and-greening-its-trade-would-offer-new

68. In India, which one of the following is responsible for maintaining price stability by controlling inflation ?

(a) Department of Consumer Affairs

(b) Expenditure Management Commission

(c) Financial Stability and Development Council

(d) Reserve Bank of India

  • Reserve Bank of India is responsible for maintaining price stability and controlling inflation.
  • It is the responsibility of MPC in India, but RBI itself plays a large role in MPC and also carries out task of monetary policy committee.
  • So RBI is the correct answer if MPC is not given in the options.

Source: Monetary Functions by RBI

69. With reference to Non-Fungible Tokens (NFTs), consider the following statements :

  • They enable the digital representation of physical assets.
  • They are unique cryptographic tokens that exist on a blockchain.
  • They can be traded or exchanged at equivalency and therefore can be used as a medium of commercial transactions.

(c) 1 and 3 only 1

  • NFTs typically contain references to digital files such as photos, videos, and audio. Because NFTs are uniquely identifiable, they differ from cryptocurrencies, which are fungible. The market value of an NFT is associated with the digital file it references.
  • Non-fungible tokens (NFTs) are cryptographic assets on a blockchain with unique identification codes and metadata that distinguish them from each other.
  • Statement 3 is incorrect : Unlike cryptocurrencies, they cannot be traded or exchanged at equivalency. This differs from fungible tokens like cryptocurrencies, which are identical to each other and, therefore, can serve as a medium for commercial transactions.
  • NFTs are unique cryptographic tokens that exist on a blockchain and cannot be replicated.
  • NFTs can represent real-world items like artwork and real estate.
  • "Tokenizing" these real-world tangible assets makes buying, selling, and trading them more efficient while reducing the probability of fraud.
  • NFTs can also function to represent individuals' identities, property rights, and more.
  • The distinct construction of each NFT has the potential for several use cases. For example, they are an ideal vehicle to digitally represent physical assets like real estate and artwork. Because they are based on blockchains, NFTs can also work to remove intermediaries and connect artists with audiences or for identity management. NFTs can remove intermediaries, simplify transactions, and create new markets.

Source: https://www.investopedia.com/non-fungible-tokens-nft-5115211

70. Consider the following pairs:

Reservoirs States

  • Ghataprabha – Telangana
  • Gandhi Sagar – Madhya Pradesh
  • Indira Sagar – Andhra Pradesh
  • Maithon – Chhattisgarh

How many pairs given above are not correctly matched ?

Only one pair is correct.

  • Ghataprabha- Karnataka
  • Gandhi Sagar- Madhya Pradesh
  • Indira Sagar – Madhya Pradesh
  • Maithon- Jharkhand

Source: Indian Geography, Indian Map

71. In India, which one of the following compiles information on industrial disputes, closures, etrenchments and lay-offs in factories  employing worker?

(a) Central Statistics Office

(b) Department for Promotion of Industry and Internal Trade

(c) Labour Bureau

(d) National Technical Manpower Information System

  • Statement ( c )  is correct : Labour Bureau has been bringing out Statistics on ?Industrial Disputes, Closures, Retrenchments and Lay-offs in India? based on the voluntary returns received every month from the Labour Departments of the States and Union Territories and the Regional Labour Commissioners

Source: labourbureaunew.gov.in

72. In India, what is the role of the Coal Controller’s Organization (CCO) ?

  • CCO is the major source of Coal Statistics in Government of India.
  • It monitors progress of development of Captive Coal/Lignite blocks.
  • It hears any objection to the Government’s notification relating to acquisition of coal-bearing areas.
  • It ensures that coal mining companies deliver the coal to end users in the prescribed time.

(b) 3 and 4 only

(c) 1 and 2 only

  • Statement 1 is correct: Coal Controller has been made the statistical authority with respect to coal and lignite statistics. Entrusted with the responsibility of carrying out Annual Coal & Lignite survey and publishing of Provisional Coal Statistics and Coal Directory of India.
  • Statement 2 is correct: Functioning of the Coal Controller's Organisation (CCO)is  entrusted with the task of monitoring captive mines.
  • Statement 3 is correct: C oal Controller is the competent authority under this act to hear any objection to the Central Government’s Notification relating to acquisition of coal bearing land and to furnish his reports to Central Govt . 

Source: coalcontroller.gov.in

73. If a particular area is brought under the Fifth Schedule of the Constitution of India, which one of the following statements best reflects the consequence of it?

(a) This would prevent the transfer of land of tribal people to non-tribal people.

(b) This would create a local self-governing body in that area.

(c) This would convert that area into a Union Territory.

(d) The State having such areas would be declared a Special Category State.

  • Statement (a) is correct: Governor can make regulations for the peace and good government of a scheduled area after consulting the tribes advisory council. Such regulations may prohibit or restrict the transfer of land by tribal to non tribal members or among members of the scheduled tribes, regulate the allotment of land to members of the scheduled tribes.
  • Statement (b) is incorrect: Tribal advisory council is an advisory body , not a governing body.
  • Statement (c) and (d) are incorrect (Self explanatory)

Source: Laxmikanth

74. Consider the following statements:

  • The India Sanitation Coalition is a platform to promote sustainable sanitation and is funded by the Government of India and the World Health Organization.
  • The National Institute of Urban Affairs is an apex body of the Ministry of Housing and Urban Affairs ‘in Government of India and provides innovative solutions to address the challenges of Urban India.
  • Statement 1 is incorrect: ISC is a multi-stakeholder platform that brings together the private sector, government, financial institutions, civil society groups, media, donors/bi-lateral/multilateral, experts etc. to work in the sanitation space to drive sustainable sanitation through a partnership model.
  • Statement 2 is correct: The National Institute of Urban Affairs (NIUA) is India’s leading national think tank on urban planning and development. As a hub for generation and dissemination of cutting-edge research in the urban sector, NIUA seeks to provide innovative solutions to address the challenges of a fast urbanising India.
  • It is against this backdrop that in 1976, NIUA was appointed as an apex body to support and guide the Government of India in its urban development plans. Since then, it has worked closely with the Ministry of Housing and Urban Affairs, alongside other government and civil sectors, to identify key areas of research, and address the lacunae in urban policy and planning. W

Source: www.niua.in

75. Which one of the following has been constituted under the Environment (Protection) Act, 1986?

(a)      Central Water Commission

(b)      Central Ground Water Board

(c)      Central Ground Water Authority

(d)      National Water Development Agency

  • Option (c)  is correct:   Central Ground Water Authority has been constituted under Section 3 (3) of the Environment (Protection) Act, 1986 to regulate and control development and management of groundwater resources in the country.

76. With reference to the “United Nations Credentials Committee”, consider the following statements:

  • It is a committee set up by the UN Security Council and works under its supervision.
  • It traditionally meets in March, June and September every year.
  • It assesses the credentials of all UN members before submitting a report to the General Assembly for approval.

(b) 1 and 3

(c) 2 and 3

  • Statement 1 is incorrect: The United Nations Credentials Committee is a committee of the United Nations General Assembly 
  • Statement 2 is incorrect: A Credentials Committee is appointed at the beginning of each regular session of the General Assembly.
  • Statement 3 is correct: Its main purpose is to report to the Assembly regarding the credentials of the body's representatives.

Source: www.un.org

77. Which one of the following statements best describes the ‘Polar Code’?

(a) It is the international code of safety for ships operating in polar waters.

(b) It is the agreement of the countries around the North Pole regarding the demarcation of their territories in the polar region.

(c) It is a set of norms to be followed by the countries whose scientists undertake research studies in the North Pole and South Pole.

(d) It is a trade and security agreement of the member countries of the Arctic Council.

  • Statement (a) is correct: P olar Code is International Code for Ships Operating in Polar Waters. The Polar Code covers the full range of design, construction, equipment, operational, training, search and rescue and environmental protection matters relevant to ships operating in the inhospitable waters surrounding the two poles

Source: www.imo.org

78. With reference to the United Nations General Assembly, consider the following statements:

  • The UN General Assembly can grant observer status to the non-member States.
  • Inter-governmental organisations can seek observer status in the UN General Assembly.
  • Permanent Observers in the UN General Assembly can maintain missions at the UN headquarters.
  • Statement 1 is correct: The United Nations General Assembly may grant non-member states, international organizations and other entities Permanent Observer Status.
  • Statement 2 is correct: General Assembly decided that observer status would be confined to States and intergovernmental organizations whose activities cover matters of interest to the Assembly. 
  • Statement 3 is correct: Permanent Observers may participate in the sessions and workings of the General Assembly and maintain missions at the UN Headquarters.

Source: www.ask.un.org

79. With reference to the “Tea Board” in India, consider the following statements:

  • The Tea Board is a statutory body.
  • It is a regulatory body attached to the Ministry of Agriculture and Farmers Welfare.
  • The Tea Board’s Head Office is situated in Bengaluru.
  • The Board has overseas offices at Dubai and Moscow.

(c)  and 4

(d) 1 and 4

  • Statement 1 is correct: The Tea Board of India is an autonomous and statutory body created under the Tea Act, 1953.
  • Statement 2 is incorrect : Board of India is a state agency of the Government of India under the control of Ministry of Commerce and Industry.
  • Statement 3 is incorrect : Headquarters is in Kolkata
  • Statement 4 is correct : Offices are located in Kolkata, London, Moscow and Dubai 

Source: www.teaboard.gov.in

80. Which one of the following best describes the term “greenwashing” ?

(a) Conveying a false impression that a company’s products are eco-friendly and environmentally sound

(b) Non-inclusion       of ecological/environmental costs in the Annual Financial Statements of a country

(c) Ignoring the disastrous ecological consequences while undertaking infrastructure development

(d) Making mandatory provisions for environmental costs in a government project/programme

  • Statement (a) is correct : Greenwashing is the process of conveying a false impression or providing misleading information about how a company's products are more environmentally sound. 
  • Greenwashing is considered an unsubstantiated claim to deceive consumers into believing that a company's products are environmentally friendly.

Source:   www.investopedia.com

81. Consider the following statements :

  • High clouds primarily reflect solar radiation and cool the surface of the Earth.
  • Low clouds have a high absorption of infrared radiation emanating from the Earth’s surface and thus cause warming effect.
  • Statement 1 is incorrect : High clouds are often thin and do not reflect very much. They let lots of the Sun's warmth in.  They radiate less energy into space than the lower, warmer clouds. Therefore, high clouds work to "trap" more energy than the low clouds.
  • Statement 2 is incorrect: Low clouds are often quite thick and reflect lots of sunlight back to space. Low clouds are excellent reflectors. But, they don't stop the longwave energy from escaping to space. Therefore, low clouds help to cool the Earth.

Source: www.nasa.gov

82. Consider the following statements:

  • Bidibidi is a large refugee settlement in north-western Kenya.
  • Some people who fled from South Sudan civil war live in Bidibidi.
  • Some people who fled from civil war in Somalia live in Dadaab refugee complex in Kenya.
  • Statement 1 is incorrect: Bidibidi Refugee Settlement is a refugee camp in northwestern Uganda.
  • Statement 2 is correct :Bidibidi is home to over 270,000 South Sudanese refugees
  • Statement 3 is correct : Dadaab camps were established 30 years ago to accommodate Somalis fleeing their country’s civil war

83. Consider the following countries

Which of the above are members of the Organization of Turkic States?    

(a) 1, 2 and 4

(c) 2 and 5

(d) 3, 4 and 5

  • The Organization of Turkic States, formerly called the Turkic Council or the Cooperation Council of Turkic Speaking States, is an international organization comprising prominent independent Turkic countries: Azerbaijan, Kazakhstan, Kyrgyzstan, Turkey and Uzbekistan.

Source: wikipedia

84. onsider the following statements

  • Gujarat has the largest solar park in India.
  • Kerala has a fully solar powered International Airport.
  • Goa has the largest floating solar photovoltaic project in India.

Which of the statements given below is/are correct?

Statement 1 is incorrect: India's Bhadla Solar Park in Rajasthan is the largest solar power park in the world.

Statement 2 is correct: Kerala's Cochin International Airport Ltd (CIAL) is the first airport in the world that would be running fully on solar power.

  • Statement 3 is incorrect: The 600 MW capacity floating solar project on Omkareshwar reservoir (in Madhya Pradesh) is not only India's but also the world's largest floating solar project so far.

https://www.saurenergy.com/solar-energy-blog/the-top-5-upcoming-floating-solar-power-projects-in-india#:~:text=The%20600%20MW%20capacity%20floating,connected%20floating%20solar%20photovoltaic%20projects .

https://www.thehindu.com/sci-tech/energy-and-environment/worlds-largest-solar-park-in-bhadla-india/article37462665.ece

https://www.livemint.com/news/india/anand-mahindra-posts-pictures-of-kochi-airport-first-in-the-world-to-be-fully-solar-powered-11638760041722.html

85. With reference to the United Nations Convention on the Law of Sea, consider the following statements:

  • A coastal state has the right to establish the breadth of its territorial sea up to a limit not exceeding 12 nautical miles, measured from baseline determined in accordance with the convention.
  • Ships of all states, whether coastal or land-locked, enjoy the right of innocent passage through the territorial sea.
  • The Exclusive Economic Zone shall not extend beyond 200 nautical miles from the baseline from which the breadth of the territorial sea in measure.
  • Statement 1 is correct : Every State has the right to establish the breadth of its territorial sea up to a limit not exceeding 12 nautical miles, measured from baselines determined in accordance with this Convention.
  • Statement 2 is correct : The innocent passage has been codified in the United Nations Convention on the Law of the Sea (UNCLOS III) was adopted in 1982, it is also known as the Law of the Sea Treaty. Its purpose is to establish a comprehensive set of rules governing the oceans and to replace previous U.N. Conventions on the Law of the Sea, 1958 (UNCLOS I) which was adopted in 1958 and another in 1960 (UNCLOS II), since these two conventions were believed to be inadequate. The right of innocent passage of foreign ships through the territorial waters of a coastal state is one of the oldest and most universally recognized rules of public international law.
  • Statement 3 is incorrect : The continental shelf and the exclusive economic zone (EEZ) are distinct maritime zones. The continental shelf includes only the seabed and subsoil; whereas the EEZ includes the water column. Also, while the maximum extent of the EEZ is 200 nautical miles, the continental shelf may extend beyond 200 nautical miles from the coastline, depending on the depth, shape, and geophysical characteristics of the seabed and sub-sea floor. The ECS is, therefore, not an extension of the EEZ. Some of the sovereign rights that a coastal State may exercise in the EEZ, especially rights to the resources of the water column (e.g., pelagic fisheries), do not apply to the ECS.

Sources: 

The Right of Innocent Passage in the UN Convention (lawteacher.net) , PREAMBLE TO THE UNITED NATIONS CONVENTION ON THE LAW OF THE SEA ,

UNITED NATIONS CONVENTION ON THE LAW OF THE SEA

https://www.state.gov/frequently-asked-questions-u-s-extended-continental-shelf-project/

86. Which one of the following statements best reflects the issue which Senkaku Islands, sometimes mentioned in the news?

(a) It is generally believed that they are artificial islands made by a country around South China Sea.

(b) China and Japan engage in maritime disputes over these islands in East China Sea

(c) A permanent American military base has been set up there to help Taiwan to increase its defence capabilities.

(d) Through International Courts of Justice declared them as no man’s land, some South-East Asian countries claim them.

Senkaku/Diaoyu islands Dispute:

Japan and China claim the uninhabited islands, known as the Senkaku in Japan and Tiaoyu in China, as their own, but Japan has administered them since 1972. The Senkaku/Diaoyu Islands were formally claimed by Japan in 1895. After Japan’s defeat in World War II, the island chain was controlled by the US until 1971 before its return. Since then, Japan has administered the island chains. China began to reassert claims over the Senkaku/Diaoyu Islands in the 1970s, citing historic rights to the area. However, Japan does not recognise Chinese claims. More recently, there has been a flare up in the region. The Japanese government said on Thursday it had protested to China regarding a set of names recently assigned by Beijing to seabed zones in the East China Sea, including the Senkaku/Diaoyu islands. 

What is the dispute about?

  • The dispute is over the claim of Senkaku islands, which have been controlled by the Japanese since 1895.
  • However, between 1945 to 1972, the islands were administered by the United States. In 1971, the US handed over the authority to Japan in 1971.
  • The island chain, claimed by China, Taiwan and Japan, is made up of five islets and three barren rocks covering an area of 7 square kilometres

Source: https://iasscore.in/rs-tv/gist-of-rajya-sabha-tv-senkaku-diaoyu-islands-dispute

87. Consider the following pairs

How many pairs given above are correcly matched?

(d) All four Pairs

Pair 1 is incorrectly matched:

general studies manual paper 1 2022 pdf

Pair 2 is correctly matched:

The president of Guinea was detained after a successful military coup in the West African nation. The coup was reportedly led by special task forces led by Colonel Mamady Doumbouya, the head of Guinean Special Forces.

Pair 3 is correctly matched:

Lebanon's economy collapsed after it defaulted on about $31 billion of Eurobonds in March 2020, with its currency sinking more than 90 per cent against the US dollar on the black market. Political bickering and indecision by the previous parliament forced the economy into a tailspin.

Inflation in the country has continued to surge and reached 206 per cent in April as the country elected a new parliament, which will have to put in place reforms to secure $3bn from the International Monetary Fund.

Lebanon’s economy contracted about 58 per cent between 2019 and 2021, with gross domestic product plummeting to $21.8bn in 2021, from about $52bn in 2019, said the World Bank.

The depression is among the world's worst economic collapses since the 1850s, the Washington lender said in January. It is the largest contraction on a list of 193 countries.

Pair 4 is correctly matched:

Tunisia's President Kais Saied on Monday extended his months-long suspension of parliament until new elections in December 2022, while calling for a July referendum on constitutional reforms.

https://www.opindia.com/2021/09/successful-military-coupe-in-guinea-government-constitution-dissolved/

https://www.thenationalnews.com/business/economy/2022/05/31/lebanon-needs-credible-reforms-to-avoid-destruction-of-economy-world-bank-says/

https://www.france24.com/en/live-news/20211213-tunisia-president-extends-parliament-suspension-sets-election-in-1-year

88. Consider the following pairs :

How many pairs given above are .correctly matched?

(d)All four pairs

  • Pair 1 is correctly matched: A broad peninsula that lies between the Black Sea and the Mediterranean Sea and called Asia Minor (Lesser Asia) by the Romans, is the Asian part of modern Turkey, across Thrace. It lies across the Aegean Sea to the east of Greece and is usually known by its ancient name Anatolia.
  • Pair 2 is correctly matched: The Amhara are one of the two largest ethnolinguistic groups in Ethiopia.
  • Pair 3 is incorrectly matched: Cabo Delgado is a region in Mozambique.
  • Pair 4 is incorrectly matched: Catalonia is a region of Spain.

https://www.allaboutturkey.com/anatolia.html

https://www.dw.com/en/ethiopias-amhara-region-shattered-after-weeks-of-war/a-60145364

https://www.msf.org/violence-cabo-delgado-many-have-seen-dead-bodies-along-way

https://www.bbc.com/news/world-europe-20345071

89. With reference to Indian laws about wildlife protection, consider the following statements:

  • Wild animals are the sole property of the government.
  • When a wild animal is declared protected, such animal is entitled for equal protection whether it is found in protected areas or outside.
  • Apprehension of a protected wild animal becoming a danger to human life is sufficient ground for its capture or killing.

Statement 1 is correct:   In a significant verdict, the Bombay High Court has ruled that wild animals including tiger should be treated as "government property for all purposes" and any damage caused by them should be compensated by the Government.

Statement 2 is correct:   The law governing the subject of wildlife, the Wildlife (Protection) Act, 1972, does not discriminate between animals found in protected areas and outside. It provides for equal protection for wild animals irrespective of where they are found.

Statement 3 is incorrect:   Only if the wild animal becomes a danger to human life or is diseased or disabled beyond recovery can it be allowed to be captured or killed by the competent authority, the Chief Wildlife Warden of the State. This provision is applicable to wild animals listed in Schedule I of the Wildlife (Protection) Act, 1972, which includes leopards.   Mere apprehension or fear that a wild animal could endanger human life is not a ground for capture or killing.

http://archive.indianexpress.com/news/now-all-wild-animals-are-govt-property/936369/#:~:text=In%20a%20significant%20verdict%2C%20the,be%20compensated%20by%20the%20Government .

https://www.thehindu.com/opinion/op-ed/Leopards-in-a-spot/article16913392.ece

90. Certain species of which one of the following organisms are well known as cultivators of fungi?

(b) Cockroach

Fungi and insects are two hyperdiverse groups of organisms that have interacted for millennia. Over time, some insects have come to rely on fungi for a variety of resources, including room and board. Ants, wasps, beetles and a variety of other insects have adapted to using fungi primarily for reinforcing structures or as sources of food, with the most extreme examples resulting in cultivation of fungal crops. Chief among these examples are the mushroom-farming ants and termites, and the wood-boring beetles and wasps.

Source:   https://onlinelibrary.wiley.com/doi/abs/10.1002/9780470015902.a0027211

91. Consider the following pairs :

How many pairs given above are correctly matched?        

  • Pair 1 is correctly matched: Dhauli is located in Odisha.
  • Pair 2 is correctly matched: Erragudi or Yerragudi is located in Andhra Pradesh
  • Pair 3 is incorrectly matched: Jaugada is located in Odisha.
  • Pair 4 is incorrectly matched: Kalsi is located in Uttarakhand.

general studies manual paper 1 2022 pdf

Source: http://www.columbia.edu/itc/ealac/landesman/summer_public_html/week1/maps/ashokamap.html https://www.worldhistory.org/Edicts_of_Ashoka/

92. Consider the following pairs:

  • Pair 1 is correctly matched: In the first quarter of the 9 th century CE, the Chandella dynasty was founded by Nannuka, who was the ruler of a small kingdom and established his capital at Kharjjuravahaka (Khajuraho).
  • Pair 2 is incorrectly matched: Jayashakti also belonged to Chandela Dynasty
  • Pair 3 is correctly matched: Nagabhatta II (c. 800–833 CE) was a king of Gurjara−Pratiharas dynasty.
  • Pair 4 is incorrectly matched: Bhoja I / MihirBhoja (c.836−885 CE) was the grandson of Nagabhatta II.

Ancient and Medieval history by Poonam Dalal Dahiya

93. Which one of the following statements about Sangam literature in ancient South India is correct ?

(a) Sangam poems are devoid of any reference to material culture.

(b) The social classification of Varna was known to Sangam poets.

(c) Sangam poems have no references to warrior ethic.

(d) Sangam literature refers to magical forces as irrational.

Sangam Literature has references to material culture and war tactics and ethics. The varna system was not practiced but it was known. 

94. “Yogavasistha” was translated into Persian by Nizamuddin Panipati during the reign of:

(b) Humayun

(c) Shahjahan

(d) Aurangzeb

YogaVasistha was translated into Persian multiple times throughout the Mughal Dynasty, as commanded by Akbar, Jahangir, and Darah Shikuh. Nizam al-Din Panipati completed one of these translations, known as the Jug-Basisht, in the late sixteenth century AD.

95. The world’s second tallest statue in sitting pose of Ramanuja was inaugurated by the Prime Minister of India at Hyderabad recently. Which one of the following statements correctly represents the, teachings of Ramanuja?

(a) The best means of salvation was devotion.

(b) Vedas are eternal, self-existent and wholly authoritative.

(c) Logical arguments were essential means for the highest bliss.

(d) Salvation was to be obtained through meditation.

  • He was born in Tamil Nadu in the eleventh century. He was deeply influenced by the Alvars. He firmly believed that intense devotion to Vishnu was the best means to attain salvation. He propounded the doctrine of Vishishtadvaita.
  • Vishishtadvaita is formed by the two words: vishisht meaning qualified and advaita meaning oneness of the individual soul and the Supreme God. In other words it means qualified oneness wherein the soul, even when united with the Supreme God, remained distinct.
  • The new strand of bhakti that developed in north India subsequently was greatly inspired by Ramanujam's doctrine.

https://www.excellup.com/sudha_r/7_history/7_history_chapter_8_2.aspx

96. The Prime Minister recently inaugurated the new Circuit House near Somnath Temple at Veraval. Which of the following statements are correct regarding Somnath Temple ?

  • Somnath Temple is one of the Jyotirlinga shrines.
  • A description of Somnath Temple was given by Al-Biruni.
  • Pran Pratishtha of Somnath Temple (installation of the present day temple) was done by President S. Radhakrishnan.
  • Somnath Temple at Somnath, Gujrat is one of the most sacred pilgrimage sites for Hindus and is believed to be first among the twelve Jyotirlinga shrines of Shiva. 
  • Ancient Indian traditions maintain a close relationship of Somnath with release of Chandra (Moon God) from the curse of his father-in-law Daksha Prajapati. Moon was married to Twenty-Seven daughters of Daksha. However, he favoured Rohini and neglected other queens. The aggrieved Daksha cursed Moon and the Moon lost power of light. With the advice of Prajapita Brahma, Moon arrived at the Prabhas Teerth and worshipped Bhagvan Shiva. Pleased with the great penance and devotion of Moon, Bhagvan Shiva blessed him and relieved him from the curse of darkness. Pauranic traditions maintain that Moon had built a golden temple, followed by a silver temple by Ravana, Bhagvan Shree Krishna is believed to have built Somnath temple with Sandalwood.
  • The later sources of history account for several desecrations by Muslims invaders during eleventh to eighteen century A.D. The temple was rebuilt every time with the reconstructive spirit of the people. The modern temple was reconstructed with the resolve of Sardar Patel who visited the ruins of Somnath temple on November 13 1947. Then President of India, Dr. Rajendra Prasad, did the Pran-Pratistha at the existing temple on 11 May 1951.

So statement 3 is incorrect.

Source: https://somnath.org/Home/Somnath-Darshan

97. Which one of the following statements best describes the role of B cells and T cells in the human body ?

(a) They protect the body from environmental allergens.

(b) They alleviate the body’s pain and inflammation.

(c) They act as immunosuppressants in the body.

(d) They protect the body from the diseases caused by pathogens.

Lymphocytes are a type of white blood cells. They play a critical role in keeping us healthy. Without them, we can’t survive.

  • T cells protect us from infection. In our daily lives, we’re constantly exposed to pathogens, such as bacteria, viruses and fungi. Without T lymphocytes, also called T cells, every exposure could be life-threatening. T cells can wipe out infected or cancerous cells. They also direct the immune response by helping B lymphocytes to eliminate invading pathogens.
  • B cells create antibodies. B lymphocytes, also called B cells, create a type of protein called an antibody. These antibodies bind to pathogens or to foreign substances, such as toxins, to neutralize them. For example, an antibody can bind to a virus, which prevents it from entering a normal cell and causing infection. B cells can also recruit other cells to help destroy an infected cell.

Source: https://www.mdanderson.org/cancerwise/t-cells--b-cells-and-the-immune-system.h00-159465579.html#:~:text=T%20cells%20can%20wipe%20out,of%20protein%20called%20an%20antibody .

98. Consider the following statements:

  • Other than those made by humans, nanoparticles do not exist in nature.
  • Nanoparticles of some metallic oxides are used in the manufacture of some cosmetics.
  • Nanoparticles of some commercial products which enter the environment are unsafe for humans.

(a) .1 only

(b). 3 only

(c). 1 and 2

(d). 2 and 3

  • Statement 1 is incorrect:   Natural nanoparticles are formed from magma spewing volcanoes;others are formed by forces such as of oceanic breakers, tides, river currents, etc.
  • Statement 2 and 3 are correct:   Among the metal oxide NPs present in cosmetic products,   amorphous silica (SiO 2 ), zinc oxide (ZnO) and particularly titanium dioxide (TiO 2 )   are the most frequent.   The thing that makes nanoparticles so interesting for applications is that materials in nanosize have properties (optical, chemical, magnetic, biological, electrical, and mechanical) that are completely different from the properties of the same material in bulk. These new properties may cause a problem if nanoparticles are unintentionally released – very few of these particles exist in nature and we as humans have not been exposed to them throughout evolution. Therefore, we cannot be sure that our body have developed defense mechanisms to deal with them.  
  •  An important consideration in the deposition of particles in the respiratory system is the lung-lining fluid (a complex mixture of lipids and surfactant proteins) since any depositing material quickly becomes coated in these surfactant proteins and lipids. Such a coating gives the deposited particle its ‘biological identity’ This so-called protein corona, which is likely to play a role in the way particles interact with lung cells such as alveolar macrophages. 
  • Several studies on workers exposed to carbon nanotubes have shown a significant increase of biomarkers of fibrosis. The organization for cancer research (IARC) of the World Health Organization (WHO) have classified one type of carbon nanotubes (Mitzui 7) as potentially carcinogenic in humans [IARC 111]. 
  • https://www.researchgate.net/publication/296485097_Natural_nanoparticles_an_overview#:~:text=Natural%20nanoparticles%20are%20formed%20from,nanoparticles%20in%20a%20living%20system .
  • https://www.sciencedirect.com/science/article/abs/pii/S0166526X21000209#:~:text=Among%20the%20metal%20oxide%20NPs,2 )%20are%20the%20most%20frequent.
  • https://www.nano.lu.se/research/nanosafety/frequently-asked-questions-about-nanosafety/faq-nano-dangerous

99. Consider the following statements:

DNA Barcoding can be a tool to :

  • assess the age of a plant or animal.
  • distinguish among species that look alike.
  • identify undesirable animal or plant materials in processed foods.

Statement 1 is incorrect:   To measure the age of plant and animal remains from the more recent past, scientists use a radioactive isotope of carbon, called carbon-14, as their clock. As carbon-14 decays, with a half-life of about 5,730 years, it becomes nitrogen-14. Using this clock, they have dated bones, campfires and other objects as old as 60,000 years, and in some cases even older.

Statement 2 and 3 is correct:   DNA barcoding uses specific regions of DNA in order to identify species. Initiatives are taking place around the world to generate DNA barcodes for all groups of living organisms and to make these data publically available in order to help understand, conserve, and utilize the world's biodiversity. For land plants the core DNA barcode markers are two sections of coding regions within the chloroplast, part of the genes, rbcL and matK. In order to create high quality databases, each plant that is DNA barcoded needs to have a herbarium voucher that accompanies the rbcL and matK DNA sequences. The quality of the DNA sequences, the primers used, and trace files should also be accessible to users of the data. Multiple individuals should be DNA barcoded for each species in order to check for errors and allow for intraspecific variation. The world's herbaria provide a rich resource of already preserved and identified material and these can be used for DNA barcoding as well as by collecting fresh samples from the wild. These protocols describe the whole DNA barcoding process, from the collection of plant material from the wild or from the herbarium, how to extract and amplify the DNA, and how to check the quality of the data after sequencing.

https://www.nist.gov/how-do-you-measure-it/how-do-you-measure-age-things#:~:text=To%20measure%20the%20age%20of,%2C%20it%20becomes%20nitrogen%2D14 .

https://www.researchgate.net/publication/267932973_DNA_barcoding_for_plants

100. Consider the following

  • Carbon monoxide
  • Nitrogen oxide
  • Sulphur dioxide

Excess of which of the above in the environment is/are cause(s) of acid rain?

(d) 1, 3 and 4

  • Acid rain results when sulfur dioxide (SO2) and nitrogen oxides (NOX) are emitted into the atmosphere and transported by wind and air currents. The SO2 and NOX react with water, oxygen and other chemicals to form sulfuric and nitric acids.  These then mix with water and other materials before falling to the ground.
  • While a small portion of the SO2 and NOX that cause acid rain is from natural sources such as volcanoes, most of it comes from the burning of fossil fuels.  The major sources of SO2 and NOX in the atmosphere are:
  • Burning of fossil fuels to generate electricity.  Two thirds of SO2 and one fourth of NOX in the atmosphere come from electric power generators.
  • Vehicles and heavy equipment.
  • Manufacturing, oil refineries and other industries.
  • Winds can blow SO2 and NOX over long distances and across borders making acid rain a problem for everyone and not just those who live close to these sources. 

Source: https://www.epa.gov/acidrain/what-acid-rain

UPSC Prelims Exam 2022 is scheduled on June 5th 2022 (Sunday). As the official answer sheet is released by   UPSC   once the final result comes out. In this scenario, all aspirants need an authentic and reliable source for making a fine assessment of marks in the  IAS Prelims exam . This assessment is quite necessary as it helps you to decide the contours of your preparation strategy.   The students remain quite apprehensive regarding their selection in prelims due to uncertain trend of cutoff in UPSC prelims.  Students who remain on the fringe of expected cut-off are often haunted by the result of UPSC Prelims.

Keeping the student’s situation in cognizance , GS SCORE   has once again come up with  the  most reliable, authentic, and accurate   Prelims Answer Key   of  UPSC Prelims 2022 . Aspirants will be able to access   IAS Prelims 2022   Answer Key   just after few hours of  Prelims General Studies Paper 1 . The Link for   UPSC Prelims Answer Key 2022   will be activated after few hours of Paper 1(General Studies).

Candidates can calculate their estimated score using the Answer Key for Prelims 2022 . The official   PT Answer Key 2022   will be released soon after the   UPSC Prelims   are conducted. The answer key will be available in PDF form for all sets – A, B, C, and D.

Candidates can use the   UPSC IAS Answer Key   for calculating their approximate score. The candidates can calculate the   UPSC CSE 2022 Prelims   marks in the Prelims using the following marking criteria: 2 Marks are awarded for each correct answer in Paper I, while 2.5 Marks are allotted in Paper II for each correct answer. However, 1/3rd of the mark allotted to a question is deducted for the wrong answer.

UPSC IAS Prelims Answer Key 2022 with explanation   will help students to know their correct and incorrect responses and they can calculate the marks that they are going to score in the Prelims exam. The final selection of the candidates for the IAS post depends on the marks secured in the UPSC Mains exam and Interview. UPSC Prelims Exam is the first step of the selection Process but the marks will not be considered while preparing the Final Merit List.

The UPSC Mains 2022 exam is scheduled to begin on September 16, 2022. The official notification for UPSC IAS 2022 was released on February 02, 2022. The UPSC will release the   UPSC Prelims 2022 Answer Key   on its official website. All those candidates who will appear for the exam can check the   IAS Prelims  Answer Key 2022   on the official website.

The candidates are informed that the various coaching institutes release the   Civil Services Prelims Answer key   on the same day of the exam. However, UPSC will release the Official answer keys once the Recruitment process is over.

GS Mains Classes

Verifying, please be patient.

Our Centers

DELHI (Karol Bagh)

GS SCORE, 1B, Second Floor, Pusa Road, Karol Bagh, New Delhi - 110005 (Beside Karol Bagh Metro Station Gate No. 8)

Get directions on Google Maps

BHUBANESWAR (Jaydev Vihar)

GS SCORE, Plot No.2298, Jaydev Vihar Square, Near HCG Day Care, BBSR - 751013

LUCKNOW (Aliganj)

GS SCORE, 2nd Floor, B-33, Sangam Chauraha, Sector H, Aliganj, Lucknow, UP - 226024

general studies manual paper 1 2022 pdf

© 2024 IAS SCORE. All Rights Reserved

general studies manual paper 1 2022 pdf

Welcome to our secure login portal. Access your account with ease.

Basix Education

  • Using Password

Not registered yet? register here!

Welcome to our secure register portal. For a brighter future, register now and unlock endless learning opportunities.

User Register

Already have an account? Login

Oops, forgot your password? Don't worry, we've got you covered. Reset it here

Lost your login details? No problem! forgot your password in just a few clicks

Forgot Password

Verify your mobile number, you have successfully logged in.

general studies manual paper 1 2022 pdf

Join Us on WhatsApp

Universal Book Seller

Add to Wishlist Add to Wishlist

general studies manual paper 1 2022 pdf

General Studies Manual Paper 1- 2022 For Civil Services Preliminary Exam Also Useful For State Public...

₹ 1,595.00 ₹ 1,276.00

Description

Additional information.

  • Reviews (0)

General Studies Manual Paper 1- 2022 For Civil Services Preliminary Exam Also Useful For State Public Service (9789354601477)

The comprehensively revised edition of General Studies Paper II, 2022, is exclusively created as per the latest UPSC Preliminary examination. This edition focuses on providing the quality content with the coverage of complete syllabus prescribed by the UPSC for Preliminary examination. This title comes fresh with thorough revision, as per the changed examination pattern with excellent quality, updated data and questions based on a carefully structured analysis of previous year’s question papers and the emerging trends. Each topic is dealt with separately, with inputs on how to prepare, supported by multiple-choice questions, with answers, for sufficient practice. This title is also useful for other State Civil Services Examination.

KEY FEATURES

• Includes June 2020 Question Paper along with Solutions and frequency analysis • Includes 150 Comprehension passages as per the latest pattern • Thoroughly revised content for logical reasoning and analytical ability with new chapters and practice questions • New examination-oriented practice questions in the General Mental Ability and Basic Numeracy & Data Interpretation sections • Thoroughly revised 15 practice test papers • Comprehensive coverage of the syllabus of UPSC other State Civil Services examinations, Accuracy of content and information with abundance of modular practice sessions

  • Publisher :
  • Langauage :
  • Publishing Date :
  • Product Editin :

There are no reviews yet.

Your email address will not be published. Required fields are marked *

Your review  *

Name  *

Email  *

Related products

general studies manual paper 1 2022 pdf

Arihant: 10 Practice Sets CDS Combined Defence Services Entrance Examination (Hindi) – Edition...

general studies manual paper 1 2022 pdf

MHE: General Studies Paper-1 NCERT for Civil Services & State Services Examinations 2nd Edition By...

general studies manual paper 1 2022 pdf

MHE: Samanya Adhyayan NCERT Vishayvar Sar Sangrah By Sheelwant Singh & Kriti Rastogi (9789387572409)

general studies manual paper 1 2022 pdf

Uttar Pradesh 2018 By Suchna Evam Jan Sampark Vibhag, U.P. (9788176783019) Hindi

Free Upsc Materials

Get Standard Books

Must have books for ias preparation.

We have Picked Some Must Have Books for IAS Preparation according to their Category.

As most of the books are available in PDF format and available in this link    Click to Download .

Following links are for people who can’t read through eBooks only  and for reading PDF you should have a Tablet or a laptop for avoiding Eye strain.

GEOGRAPHY [NCERT BOOKS AND GC LEONG ARE ENOUGH]

CULTURE [NITIN SINGHANIA ENOUGH]

POLITY AND ECONOMY

ENVIRONMENT

SCIENCE AND TECHNOLOGY

Most Important and Additional IAS Preparation Books for UPSC Mains (Click to Buy Online)

Short Notes IAS PQRN 2023

Join Our Telegram Channel

IAS Preparation Books FOR GENERAL STUDIES PAPER 1

IAS Preparation Books FOR GENERAL STUDIES PAPER 2

IAS Preparation Books FOR GENERAL STUDIES PAPER 3

IAS Preparation Books FOR GENERAL STUDIES PAPER 4

IAS Preparation Books for Interview

Must Get UPSC IAS Practice Books available in Amazon

General Studies Manual (GS Manual)

GS manual should be with you when you are going to prepare for the prelims examination. Now arises the question that what is GS manual? It’s an all-in-one book for civil services prelims covering all the topics like Polity, Economy, History, Geography, Current Affairs, General Knowledge, Mental Ability, Facts, and more. One Book in hand with all the information you need is like the god on the earth for IAS examination Aspirants. Some times standard books prescribed for preparation do not have that information which GS manual have. Above that, the language and content are so easy to understand for any of the aspirants. The level set is also satisfactory.

The passed candidate suggests that one must go through the past 5 years Solved Section-wise Papers and IAS Syllabus before going to any other books. These papers will be a great boon for IAS aspirants. Valuable time and efforts are to be used in the right place so these two reference books will be best for those who start preparing for Prelims. They must be sure to get the solved section-wise questions papers and the IAS syllabus.

Receive the latest articles in your inbox

Email Address

Subscribe Now

U.S. flag

A .gov website belongs to an official government organization in the United States.

A lock ( ) or https:// means you've safely connected to the .gov website. Share sensitive information only on official, secure websites.

  • Handwashing
  • Hand Hygiene as a Family Activity
  • Hand Hygiene FAQs
  • Handwashing Facts
  • Publications, Data, & Statistics
  • Health Promotion Materials
  • Global Handwashing Day
  • Life is Better with Clean Hands Campaign
  • Clinical Safety
  • Healthcare Training
  • Clean Hands Count Materials

About Handwashing

  • Many diseases and conditions are spread by not washing hands with soap and clean, running water.
  • Handwashing with soap is one of the best ways to stay healthy.
  • If soap and water are not readily available, use a hand sanitizer with at least 60% alcohol to clean your hands.

boy showing his clean hands

Why it's important

Washing hands can keep you healthy and prevent the spread of respiratory and diarrheal infections. Germs can spread from person to person or from surfaces to people when you:

  • Touch your eyes, nose, and mouth with unwashed hands
  • Prepare or eat food and drinks with unwashed hands
  • Touch surfaces or objects that have germs on them
  • Blow your nose, cough, or sneeze into hands and then touch other people's hands or common objects

Key times to wash hands

You can help yourself and your loved ones stay healthy by washing your hands often, especially during these key times when you are likely to get and spread germs:

  • Before, during, and after preparing food
  • Before and after eating food
  • Before and after caring for someone at home who is sick with vomiting or diarrhea
  • Before and after treating a cut or wound
  • After using the toilet
  • After changing diapers or cleaning up a child who has used the toilet
  • After blowing your nose, coughing, or sneezing
  • After touching an animal, animal feed, or animal waste
  • After handling pet food or pet treats
  • After touching garbage

How it works

Washing your hands is easy, and it’s one of the most effective ways to prevent the spread of germs. Follow these five steps every time.

  • Wet your hands with clean, running water (warm or cold), turn off the tap, and apply soap.
  • Lather your hands by rubbing them together with the soap. Lather the backs of your hands, between your fingers, and under your nails.
  • Scrub your hands for at least 20 seconds . Need a timer? Hum the “Happy Birthday” song from beginning to end twice.
  • Rinse your hands well under clean, running water.
  • Dry your hands using a clean towel or an air dryer.

Use hand sanitizer when you can't use soap and water

Washing hands with soap and water is the best way to get rid of germs in most situations. If soap and water are not readily available, you can use an alcohol-based hand sanitizer that contains at least 60% alcohol. You can tell if the sanitizer contains at least 60% alcohol by looking at the product label.

What you can do

CDC has health promotion materials to encourage kids and adults to make handwashing part of their everyday lives.

  • Share social media graphics and messages.
  • Print stickers and place clings on bathroom mirrors.
  • Promote handwashing on or around Global Handwashing Day , celebrated each year on October 15.
  • Distribute fact sheets to share information about hand hygiene for specific audiences.
  • Frequent Questions About Hand Hygiene
  • Hand Hygiene in Healthcare Settings
  • The Life is Better with Clean Hands Campaign

Clean Hands

Having clean hands is one of the best ways to avoid getting sick and prevent the spread of germs to others.

For Everyone

Health care providers.

IMAGES

  1. General Studies Manual Paper 1- 2022 For Civil Services Preliminary

    general studies manual paper 1 2022 pdf

  2. Arihant General Studies Paper-1 PDF [2022 Edition]

    general studies manual paper 1 2022 pdf

  3. Answer Key Open Discussion : Series A B C D Upsc Prelims 2022 Gs

    general studies manual paper 1 2022 pdf

  4. Answer Key Open Discussion : Series A B C D Upsc Prelims 2022 Gs

    general studies manual paper 1 2022 pdf

  5. Best UPSC GENERAL STUDIES Manual 2022 by MC Grow Hills

    general studies manual paper 1 2022 pdf

  6. Buy General Studies Manual Paper-1 2022 for Civil Serives Preliminary

    general studies manual paper 1 2022 pdf

VIDEO

  1. 25 Years UPSC CSE Prelims PYQs

  2. GENERAL EDUCATION SOCIAL SCIENCE & RESEARCH 2024 DRILLS FOR MARCH LET REVIEW DRILLS

  3. General studies (40 marks) test-02,03&04 Maths Solutions| Maths solved paper

  4. UKPSC Upper Prelims-2024

  5. UPSC Mains 2022

  6. Class 12 English Important Questions 2024

COMMENTS

  1. General Studies Manual Paper-1 2022

    1. General Studies Paper - 1 is the best- selling book particularly designed for the civil services Preliminary examinations. 2. This book is divided into 6 major sections covering the complete syllabus as per UPSC pattern 3. Special Section is provided for Current Affairs covering events, Summits and Conferences 4. simple and lucid language used for better understanding of concepts 5. 5 ...

  2. [Download] UPSC Mains General Studies Paper-1 (2013-2023 ...

    Appendix: Syllabus of UPSC Mains-GSM1 General Studies Paper1; Prologue. In 2013, UPSC changed the syllabus-pattern of Mains examination and the number of general studies papers were increased from two to four. Out of them, GS Paper-I deals with History, Culture, Society and Geography. Overall breakup looks like this

  3. Arihant General Studies Paper-1 PDF [2022 Edition]

    Arihant General Studies Manual Paper 1 Pdf. The General Studies Paper - 1 by Arihant is a highly popular book that is specifically created for the civil services Preliminary exams. It comprises 6 main sections that cover the entire UPSC syllabus. Additionally, there is a dedicated section that covers current affairs, such as conferences and ...

  4. GENERAL STUDIES Manual 2022 |GS Paper 1

    "General Studies Paper I 2022 is a thoroughly updated and comprehensive study material with a dedicated focus on the latest syllabus prescribed by the UPSC for Preliminary Examination. This revised annual edition employs coverage of the contemporary issues and developments.

  5. UPSC Prelims 2022 General Studies Question Paper 1 [Download PDF]

    The Question Paper of UPSC Prelims GS 1 2022 is available to download. The aspirants can download the PDF of the IAS Prelims 2022 General Studies-I Question Paper, using the link, given below. The Prelims exam result will be declared after a month of the exam, the eligible candidates who would have cleared the Prelims 2022 will have to fill DAF ...

  6. General Studies Manual Paper-1 2022 eBook

    1. General Studies Paper - 1 is the best- selling book particularly designed for the civil services Preliminary examinations. 2. This book is divided into 6 major sections covering the complete syllabus as per UPSC pattern 3. Special Section is provided for Current Affairs covering events, Summits and Conferences 4. simple and lucid language used for better understanding of concepts 5. 5 ...

  7. Download UPSC IAS Mains 2022 "General Studies (GS Paper 1)" Exam

    Exam Date: 17-09-2022. Subject: Paper - II : General Studies-I : (Indian Heritage and Culture, History and Geography of the World and Society) 1. How will you explain that medieval Indian temple sculptures represent the social life of those days ? (150 Words, 10 Marks) 2.

  8. Buy General Studies Manual Paper-1 2022 for Civil Serives Preliminary

    "1. General Studies Paper - 1 is the best- selling book particularly designed for the civil services Preliminary examinations. 2. This book is divided into 6 major sections covering the complete syllabus as per UPSC pattern 3. Special Section is provided for Current Affairs covering events, Summits and Conferences 4. simple and lucid language used for better understanding of concepts 5. 5 ...

  9. General Studies Manual Paper-1 2022 for Civil Serives ...

    Practice Questions provides Topicwise Questions and Previous Years Solved Papers With our all time best selling edition of General Studies Manual Paper 1 is a guaranteed success package which has been designed to provide the complete coverage to all subjects as per prescribed pattern along with the updated and authentic content.

  10. General Studies for Civil Services Preliminary Examination (Paper -1

    Key Highlights of General Studies Paper 1 for Civil Services Preliminary Examination. Simple, easy-to-read, and concise. Due to separate binding, these books are very handy, unlike other all-in-one heavy GS manuals. Macro and micro analysis of the UPSC Civil Service Exam Question paper trends - highly useful for beginners and veterans to ...

  11. GENERAL STUDIES Manual 2022 |GS Paper 1

    General Studies Paper I 2022 is a thoroughly updated and comprehensive study material with a dedicated focus on the latest syllabus prescribed by the UPSC for Preliminary Examination. This revised annual edition employs coverage of the contemporary issues and developments.

  12. UPSC CSE Mains 2022 GS Paper I: GENERAL STUDIES PAPER-I PDF Download

    UPSC 2022 Mains GS Paper 1. The General Studies Paper I consists of 20 descriptive type questions on topics such as Indian Heritage and Culture, History and Geography of the world as well as Society. This paper is worth 250 marks and has a time limit of 3 hours. This paper is being considered for merit ranking.

  13. Insights Ias

    UPSC MAINS GENERAL STUDIES PAPER - 1 MAINS 2022 GENERAL STUDIES. PAPER - 1 . 1. How will you explain the medieval Indian temple sculptures represent the social life of those days? (Answer in 150 words) 10. 2. Why did the armies of the British East India Company - mostly comprising of Indian soldiers - win consistently against the more ...

  14. UPSC CSE Mains 2022 GS Paper I: GENERAL STUDIES PAPER-I PDF Download

    UPSC CSE Mains 2022 GS Paper I: GENERAL STUDIES PAPER-I PDF Download. GENERAL STUDIES. PAPER - 1. 1. How will you explain the medieval Indian temple sculptures represent the social life of those days? (Answer in 150 words) 10. 2. Why did the armies of the British East India Company - mostly comprising of Indian soldiers - win consistently ...

  15. General Studies Manual Paper-1 2022 (For 2024 exam)

    General Studies Manual Paper-1 2022 By Arihant Publications Author(s) : Deepika Singla,Rakesh Kumar Roshan,Dr. Priya Goel,Jenmenjay Sahni,Vivek Singh,Tushar Shukla,Amibh Ranjan,Ekta Soni,Sahana Ansari,Dr Poonam Singh

  16. General Studies (Paper 1) With Solution UPSC Mains 2022: Download

    UPSC Prelims Exam 2022 was held on 5th, June 2022 (Sunday). As the official answer sheet is released by UPSC once the final result comes out.In this scenario, all aspirants need an authentic and reliable source for making a fine assessment of marks in the IAS Prelims exam.This assessment is quite necessary as it helps you to decide the contours of your preparation strategy.

  17. General Studies Manual Paper 1- 2022 For Civil Services Preliminary

    General Studies Manual Paper 1- 2022 For Civil Services Preliminary Exam Also Useful For State Public Service (9789354601477) OVERVIEW. The comprehensively revised edition of General Studies Paper II, 2022, is exclusively created as per the latest UPSC Preliminary examination. This edition focuses on providing the quality content with the ...

  18. General Studies Manual Paper-1 2022 for Civil Serives Preliminary Exam

    General Studies Manual Paper-1 2022 for Civil Serives Preliminary Exam. USD $ 96 70. Price when purchased online ... Questions provides Topicwise Questions and Previous Years' Solved PapersWith our all time best selling edition of "General Studies Manual Paper 1" is a guaranteed success package which has been designed to provide the ...

  19. Prelims

    Here, Complete E-books for Prelims GS Paper II are available for Download. 26 Years UPSC Previous Papers Solved MCQ by Disha Publication 11th Edition PDF. UPSC Prelims Gs Paper 2 CSAT Practice MCQ by Disha Publication. 22 Years Previous Papers Solved MCQ by Disha Publication 7th Edition PDF. Disha Publications 101 Speed Tests.

  20. General Studies Paper I 2024

    General Studies Paper I 2024. OverviewQualitatively restructured content in precise and crisp form, updated facts and data along with questions based on structured analysis from static to dynamic linking concepts and emerging trends on these aspects, presenting MHE's most celeb…. Read More.

  21. Get Standard Books

    TMH (GS - Paper 1) General Studies: Paper II (2020) TMH (CSAT - Paper 2) ... General Studies Manual (GS Manual) ... Vision IAS Prelims 2024 Abhyaas Test 1 Hindi Medium PDF April 27, 2024; Vision IAS Prelims 2024 Test 35 with Solutions PDF [FLT-10] [All Complete] ...

  22. Prelims-GS

    Prelims-GS. Overview:In this KPSC General Study Manual all the topics are explained in a precise and crisp format. Qualitatively recreated content, adapted to updated facts and data. Concepts are simplified, with links from static to dynamic. Questions based on structural analysis of emerging trends, presented ….

  23. About Handwashing

    Washing your hands is easy, and it's one of the most effective ways to prevent the spread of germs. Follow these five steps every time. Wet your hands with clean, running water (warm or cold), turn off the tap, and apply soap. Lather your hands by rubbing them together with the soap. Lather the backs of your hands, between your fingers, and ...